You are on page 1of 44

LEGALEDGE TEST SERIES

Part of the Most Comprehensive & Consistently Successful Study Material & Test Series Module, spanning across
both Physical and Online Programs in the entire Country. As a result, LegalEdge was able to engineer Clean-Sweep-
Landslide figures of a handsome 64 Selections under 100 ranks, and a whopping 273 selections under 500 ranks in
CLAT 2021. With AILET being no different, a total of 34 of our students found their way into NLU, Delhi in 2021.
In a nutshell, every second admit in a Top National Law School in 2021 came from the LegalEdge Preparation
Ecosystem.

MOCK COMMON LAW ADMISSION TEST 2023


MOCK CLAT - 21

Duration : 120 Minutes Candidate Name : _____________

c o m Scan this code after the test

Max. Marks
Centre Name
: 150
: __________
Batch
Contact No.
:
:
_____________
_____________

rs .
k e
r an
p
to punch in your answers
INSTRUCTIONS TO CANDIDATES (Test ID: 2529975)

1.
2.
3.
To
No clarification on the question paper can be sought. Answer the questions as they are.
There are 150 multiple choice objective type questions.
There is negative marking of 0.25 for every incorrect answer. Each question carries ONE mark. Total marks are
150
4. You have to indicate the correct answer by darkening one of the four responses provided, with a BALL PEN
(BLUE OR BLACK) in the OMR Answer Sheet.
Example: For the question, "Where is the Taj Mahal located?", the correct answer is (b).
The student has to darken the corresponding circle as indicated below:
6I4A
(a) Kolkata (b) Agra (c) Bhopal tr-5H7G4D3H1K(d) Delhi
Right Method Wrong Methods

5. Answering the questions by any method other than the method indicated above shall be considered incorrect and
6N4O
no marks will be awarded for the same. tr-5L7S4T3M1M
6. More than one response to a question shall be counted as wrong.
7. Do not write anything on the OMR Answer Sheet other than the details required and, in the spaces, provided for.
8. You are not required to submit the OMR Answer Sheet and Test Paper after the test.
9. The use of any unfair means by any candidate shall result in the cancellation of his/her candidature.
10. Impersonation is an offence and the student, apart from disqualification, may have to face criminal prosecution.
11. You have to scan the QR code only after completion of offline test.
12. You cannot leave the examination hall without punching your answers on the portal.
SECTION-A: ENGLISH LANGUAGE

Directions (Q.1-Q.30): Read the following passage carefully and answer the questions that follow.

Directions (Q.1-Q.5): Read the passage carefully and answer the questions that follow.
Financial assistance to the world's 83 weakest economies fell by 15 per cent in 2020, to $35 billion as a direct
result of the Covid-19 pandemic, UN trade and development experts UNCTAD said on Monday. According to
UNCTAD's World Investment Report 2021, total foreign direct investment also dropped by more than a third
globally, to $1 trillion (from $1.5 trillion in 2019), threatening progress on sustainable development. This level
was last seen in 2005 and it is an urgent problem because foreign direct investment is vital to promoting
sustainable development in the world's poorest regions, said Isabelle Durant, Acting Secretary-General of
UNCTAD. "The (Covid-19) crisis has had an immense negative impact on the most productive types of

m
investment, namely, greenfield investment in industrial and infrastructure projects", she said. "This means that
international production, an engine of global economic growth and development, has been seriously affected."

.c o
Regionally, Europe saw foreign direct investment fall 80 per cent last year, while flows to North America fell
by 42 per cent, which was attributed to a fall in reinvested earnings. Other developed economies saw an average

rs
drop of 20 per cent, UNCTAD said, while the African continent saw a 16 per cent fall in foreign direct investment
- to $40 billion - a level is last seen 15 years ago. Significantly, greenfield project announcements in Africa also

k e
tumbled 62 per cent, hurting industrialisation prospects, and commodity-exporters were the worst-hit. By
contrast, foreign direct investment to developing Asia resisted the worst impacts of the pandemic, driven by

n
China, where capital inflows increased by six per cent, to $149 billion. Southeast Asia saw a 25 per cent decline

a
but investment in India increased, driven in part by mergers and acquisitions. This contrasts with Latin America

r
and the Caribbean, where foreign direct investment plummeted last year, falling by 45 per cent to $88 billion.
"Many economies on the continent, among the worst affected by the pandemic, are dependent on investment in

o p
natural resources and tourism, both of which collapsed", UNCTAD said. Although foreign investment between
wealthier nations fell most in 2020 - by 58 per cent - developing nations have borne the brunt of last year's overall
investment downturn, UNCTAD said. To highlight this, the UN body pointed to the 42 per cent fall in the number

T
of new greenfield projects among fragile economies and a 14 per cent fall in international project finance deals;
the latter are significant because they drive infrastructure growth.

Looking ahead, Ms. Durant insisted that although governments were rightly focusing on shaking off the impacts
of the pandemic, the real challenge is "not only about reigniting the economy, it is about making the recovery
more sustainable and more resilient to future shocks".

1. What can be concluded from the passage? tr-5H7G4D3H


1K6I4A
(a) Global foreign direct investment flows should not be given so much importance because of their fluctuations
leading to a downfall in the economy in times of emergencies.
(b) Pandemic rolled back sustainable development funding which not only needs to be reinstated but also needs
to be made shockproof for the future.
(c) Greenfield investment is not important for developing countries as much as they are for developed countries.
4T3M1M6N4O
(d) Cross-border mergers and acquisitions are not beneficial. tr-5L7S

2. As per the passage, which of the following drives infrastructure growth?


(a) Commodity exports and international financial projects.
(b) Reinvested earnings and commodity exports.
(c) Greenfield projects and reinvested earnings.
(d) International financial projects.

Head Office: 127, Zone II, MP Nagar, Bhopal |+91-7676564400| https://www.toprankers.com Page 2 of 44
3. Which of the following words can replace the highlighted word 'PLUMMETED' as used in the passage?
(a) Increased (b) Backfire (c) Expanded (d) Plunged

4. How the pandemic has impacted the economies?


(a) Weak economies found alternatives for sustainability.
(b) Pandemic has threatened the progress of sustainable development.
(c) Foreign direct investments remain unaffected by the pandemic.
(d) Pandemic has rigorously shot up sustainable development.

5. What do you understand by the phrase "governments were rightly focusing on shaking off the impacts of the
pandemic"?

m
(a) The pandemic has shaken the global economy.
(b) Governments of pandemic hit countries know that they can't fight it back.

(d) Pandemic hit countries tried to reduce its impact on the economy front.

.c o
(c) Countries that are impacted by the pandemic haven't take any appropriate measure to deal with it.

rs
Directions (Q.6-Q.10): Read the passage carefully and answer the questions that follow.

k e
Almost 700 years ago, the overwhelmed physicians and health officials fighting a devastating outbreak of
bubonic plague in medieval Italy had no notion of viruses or bacteria but they understood enough about the Black

n
Death to implement some of the world’s first anti-contagion measures.

r a
Starting in 1348, soon after the plague arrived in cities like Venice and Milan, city officials put emergency public
health measures in place that foreshadowed today’s best practices of social distancing and disinfecting surfaces.

p
To
“They knew that you had to be very careful with goods that are being traded, because the disease could be spread
on objects and surfaces, and that you tried your best to limit person-to-person contact,” says Jane Stevens
Crawshaw a senior lecturer in early modern European history at Oxford Brookes University.

The Adriatic port city of Ragusa (modern-day Dubrovnik) was the first to pass legislation requiring the
mandatory quarantine of all incoming ships and trade caravans in order to screen for infection.

The order, which miraculously survived in the Dubrovnik archives, reads that on July 27, 1377, the city’s Major
Council passed a law “which stipulates that those who come from plague- infested areas shall not enter [Ragusa]
or its district unless they spend a month on the islet of Mrkan
tr-5H7Gor in1Kthe
4D3H 6I4Atown of Cavtat, for the purpose of
disinfection”.

Mrkan was an uninhabited rocky island south of the city and Cavtat was situated at the end of the caravan road
used by overland traders en route to Ragusa. writes Ziata Blazina Tomic in Expelling the Plague: The Health
Office and the Implementation of Quarantine in Dubrovnik, 1377-1533.1M6N4O
tr-5L7S4T3M

Tomic says that some medical historians consider Ragusa’s quarantine edict one of the highest achievements of
medieval medicine. By ordering the isolation of healthy sailors and traders for 30 days, Ragusan officials showed
a remarkable understanding of incubation periods. New arrivals might not have exhibited symptoms of the
plague, but they would be held long enough to determine if they were in fact disease-free.

The 30-day period stipulated in the 1377 quarantine order was known in Italian as a trentino, but Stevens
Crawshaw says that doctors and officials also had the authority to impose shorter or longer stays. The English
word ‘quarantine’ is a direct descendent of quarantino, the Italian word for a 40-day period.

Head Office: 127, Zone II, MP Nagar, Bhopal |+91-7676564400| https://www.toprankers.com Page 3 of 44
Why 40 days? Health officials may have prescribed a 40-day quarantine because the number had great symbolic
and religious significance to medieval Christians. When God flooded the Earth, it rained for 40 days and 40
nights, and Jesus fasted in the wilderness for 40 days.

Stevens Crawshaw says that even before the arrival of the plague, the biblical notion of a 40-day period of
purification had crossed over into health practices.

Even with the new quarantine law, Ragusa continued to be hit hard by aftershock outbreaks of the plague in 1391
and 1397. As a maritime city that survived on trade, it would have been impossible to completely wall off Ragusa
to disease without gutting the economy.

But even if the quarantine measures didn’t fully protect Ragusans from disease, Stevens Crawshaw believes that

m
the laws may have served another purpose—restoring a sense of order.

6. The structure of the passage can be best described as?


(a) The passage provides a thesis on how the concept of quarantine came into being.

.c o
(b) The passage highlights the situation of plaque during the 1 300s and how Ragusa tried to fight it.

rs
(c) The passage talks about the efforts of Ragusa in curbing the onset of plaque.
(d) The passage provides a brief history of Ragusa’s fight against plaque and the inception of the word
Quarantine.

k e
n
7. According to the passage, why is Ragusas quarantine edict considered one of the highest achievements of

a
medieval medicine?

r
(a) Because back in those times, it was remarkable to know the importance of isolation.
(b) Because isolation was introduced for the first time during this period.

o p
(c) Because Ragusa was responsible for the inception of the word ‘quarantine’.
(d) Because Ragusa could sustain the onset of plaque and protect its people.

8.

T
Which of the following cannot be concluded from the passage above?
(a) Mrkan was an uninhabited island.
(b) Irrespective of the incubation, it was impossible to keep Ragusa disease free.
(c) Zlata Blazina Tomic was responsible for the safety of the city of Ragusa.
(d) Doctors and officials also had the authority to impose shorter or longer stays.

9. Which of the following is incorrect in regards to Stevens Crawshaw’s thoughts?


(a) He believed that Ragusa was well aware of the various ways in4Dwhich
tr-5H7G the disease could spread.
3H1K6I4A
(b) He was of the opinion that Ragusa’s quarantine edict was one of the highest achievements of medieval
medicine.
(c) He believed the law helped instill the importance of order in Ragusa.
(d) 40-day period of biblical notion was passed on the health front much before.
tr-5L7S 4T3M1M6N4O
10. Which of the following options can be inferred about the author?
(a) He is a historian, interested in the opinions of Stevens Crawshaw.
(b) He is the author of a textbook that highlights various medical illnesses.
(c) He is a historian, who wishes to highlight the quarantine situation back in the 1300s.
(d) He is a scientist, trying to understand the medical sciences of the medieval period.

Head Office: 127, Zone II, MP Nagar, Bhopal |+91-7676564400| https://www.toprankers.com Page 4 of 44
Directions (Q.11-Q.15): Read the passage carefully and answer the questions that follow.
Presidential campaigns are built on webs of contingency. If Joe Biden’s candidacy had failed at some stage, Amy
Kobuchar or Pete Buttigieg might have run the table against Bernie Sanders. Or Sanders might have run the table
against one of them. Or Mike Bloomberg might have gained the Democratic Party’s nomination.
But US Representative James Clyburn’s speech endorsing Joe Biden ten days ago in the South Carolina primary
established a two-person race practically overnight and vaulted Biden into the lead. “I know Joe.” Clyburn said.
“We know Joe. Most important. Joe knows us.” To better understand how eleven words could have been so
powerful, I turned to The Age of Entitlement: America since the Sixties, by Christopher Caldwell.

Caldwell. a contributing editor to the Claremont Review of Books and an occasional opinion columnist for The
New York Times, is one of a handful of authors I follow hoping to understand what has happened to the
Republican Party. Myriad streams of opinion influence Republican position-taking, of course. I am interested

m
mainly in those that can be described as revisionist history.
They don’t come much more revisionist than Caldwell. Globalization and technology have little do with the

Rights Act of 1964.

.c o
polarization of the present day, according to him. The vile mood stems instead from the passage of the Civil

rs
What was intended to be “a transitional measure leading to a stable. racially-mixed society,” metastasized into
the template of a bill of rights for women, immigrants, LGBTs, the handicapped, the aged, and environmentalists.

k e
Caldwell says. Courts and bureaucracies have replaced democracy. The ideology of civil rights, relabelled human
rights, hardened into a body of legislation and case law that today amounts to a second constitution, according

n
to Caldwell, at odds with the version of 1788.

r a
Those who lost most from the new rights-based identity politics were white men, he writes, because the new
laws helped everybody but them. They fell asleep thinking of themselves as the people who had built this country,

p
and woke up to find themselves occupying the bottom rung of an official hierarchy of races.”

o
In this telling, the Fourteenth Amendment, with its guarantees of equal protection and due process under the law,
was a bridge too far passed in the wake of Abraham Lincoln’s assassination. The Thirteenth Amendment,

T
abolishing slavery would have been enough. The Supreme Court’s decision in Brown vs. Board of Education. in
1954, was a mistake, in that it “put certain public bodies under surveillance for racism.” Rosa Parks was not a
weary seamstress looking for a place to sit down on a bus; she was trained agitator, an intellectual leader of the
Montgomery. Ala., chapter of the NAACP, a soldier in what Harry Kalven. Jr, a long-ago University of Chicago
professor of law, described as “an almost military assault on the constitution.” Robert Bork was “a towering
figure in American legal philosophy” for expressing his misgivings about the constitutionality of civil rights
legislation. And 1992 presidential candidate Pat Buchanan was a seer for having run the first campaign against
globalization. tr-5H7G4D3H
1K6I4A

11. What is the passage about?


(a) The passage aims to highlight the differences of what a democracy should be and what a democracy currently
is, in America.
(b) The passage tries to understand the contemporary political scenario through the point of view of ‘The Age
6N4O
of Entitlement: America since the Sixties’. tr-5L7S4T3M1M
(c) The passage aims to bring to light the various political changes that have taken place from the past to the
present in America.
(d) The passage focusses on the struggle of the Blacks in the new democratic world.
12. Which of the following is LEAST likely to be agreed upon by the author?
(a) The author believes that the political scenario can be best understood by analysing Caldwell’s opinions.
(b) The author is of the opinion that the democratic situation in America is the result of some undesirable
decisions made over the course of time.
(c) The author believes that the political situation in America is very competitive in nature.
(d) The author believes that it was correct to put certain public bodies under surveillance for racism.
Head Office: 127, Zone II, MP Nagar, Bhopal |+91-7676564400| https://www.toprankers.com Page 5 of 44
13. What do you mean by the idiom ‘a bridge too far’ as used in the passage?
(a) The act of building a bridge for a very long period of time.
(b) The act of building a bridge as per the needs of equality.
(c) The act of getting into a situation which causes a lot of trouble.
(d) The act of overreaching and getting yourself into trouble.

14. Why were the white men most affected due to identity politics?
(a) White men belief that the rights-based identity politics would not hamper their position was a fallacy.
(b) White men believed that the Blacks would be ill-treated without a doubt.
(c) Identity politics refused to consider the demands of the White men.
(d) White men were denied their rights which made them reach the bottom of the pyramid.

15.
contingency’?

c o m
Which of the following can be a possible explanation for the line ‘Presidential campaigns are built on webs of

.
(a) The author believes that the presidential campaigns can be altered as per needs
(b) The author thinks that at one point of time multiple candidates can come to the fore front for presidency

rs
(c) The author believes that there is always a backup in regards to the presidential campaigns
(d) The author feels that the Presidential campaigns are built on intertwined webs, which are very difficult to
understand

k e
n
Directions (Q.16-Q.20): Read the passage carefully and answer the questions that follow.

r a
If you’re lucky enough to eke out a piece of the pie and end up working in Canadian cultural industries, you’re
in for a surprise. In the halls of studios and newsrooms and production offices, a different principle reigns: for

p
talented careerists, real success means going to America. Go to America, veterans will tell their younger

o
colleagues, held in place themselves by property or kids or the more mundane ballasts of habit and time. Get out
while you still can. The prospect of my moving to America had been coming up for years, resurfacing across my

T
work in various fields and usually raised by people other than me.
As a voice actor, I travelled to LA for awards shows and met with directors who told me, correctly, that I’d never
get cast in a blockbuster from Canada. As a law student, I was urged to apply to New York firms for better
money and job prospects. As a fact checker and editor floating between contract gigs, my situation mirrored
what I heard dispensed as common knowledge: there’s not much here. No matter the context, the drag to go to
America was present and forceful, a density in the atmosphere.
Instead, I stayed because staying was comfortable. I also resented being told by other people to take my life, tear
it up, and start again. If the factor limiting my potential wastr-truly
5H7G4Djust a 6Ifluke
3H1K 4A of birth and borders, the mistake
didn’t seem like mine to pay for, especially if the price was to uproot my whole existence. Yet, at the same time,
I felt the nagging worry that these people were right. Their advice set churning the same gut-level dread.
The real reason I never leapt at the advice to hightail it out is because I was afraid. My stubbornness didn’t bloom
from any lack of want but from an excess of it, desire at a scale I distrusted my ability to control. Or so I told
myself, since wanting it too much sounds good than crushing fear of failure. 6N4O
For all of my complaints, I was
tr-5L7S4T3M1M
content to swim in a pond of this exact circumference. I praised the people who got out but wasn’t sure I had it
in me to be one of them. That anxiety is a potent indicator: my most powerful attractions have been threaded
with the suspicion that I’m not quite good enough to merit the object of my desire. Which may be the most
Canadian thing I’ve ever said. But in some areas, Canada was superior. Free health care. Better gun control. Less
money, sure, but also less competition. What if you uproot your life, head south, and discover, talent
notwithstanding, that you’re just too soft for it?

Head Office: 127, Zone II, MP Nagar, Bhopal |+91-7676564400| https://www.toprankers.com Page 6 of 44
16. Which of the following statements are TRUE in regard to the fact that America is considered superior to Canada
in terms of employment prospects?
A. When the author travelled to Los Angeles and spoke with directors, they used to inform the author that
they'd never be cast in a Canadian blockbuster.
B. The author’s boss, after seeing the author’s potential, candidly told them to find better job opportunities in
America as they would get more value there.
C. As a law student, the author was encouraged to apply to New York law firms for greater pay and job
opportunities.
(a) Only A and B
(b) Only B and C
(c) Only A and C

m
(d) All A, B and C

17.
America?

.c o
Which of the accompanying is NOT one of the main reasons for which the author didn’t want to move to

(a) The author’s stubbornness to not moving to America grew not from a lack of want, but from an excess of

rs
it, desire on a scale she doubted she could control.
(b) The author was actually terrified to move there.

k e
(c) The author was doubtful of her merit required to make it in a foreign country like America.
(d) The author wanted to prove that moving to America was actually a result of escapism that the other people

n
were following.

18.

r a
Which of the accompanying is NOT true in reference to the passage?
(a) For years, the possibility of the author’s going to America had surfaced, emerging across her work in
numerous disciplines.

p
of opportunities.
(d) All are true. To
(b) A separate premise governs the hallways of studios, newsrooms, and production offices that true success
for talented careerists means moving to America.
(c) Veterans will encourage their younger colleagues to stay in Canada and earn a living as there are a plethora

19. Which one of the points is NOT a factor of Canada that makes it a better country than America?
(a) Free Medical Care.
(b) Gun laws.
(c) Less competition, though there is less money. tr-5H7G4D3H
1K6I4A
(d) Better sporting opportunities.

20. Which of the accompanying is NOT a conclusion that can be drawn from the passage regarding the author’s
decision to stay in Canada?
(a) The author believed that if the element restricting her potential was1Mactually
6N4O
a result of her birth and
tr-5L7S4T3M
borders, the error wasn’t something that was supposed to be paid for by her.
(b) The author was confident about her decision to stay in Canada and make a life and didn’t consider the
opinion of others.
(c) The author didn’t want to uproot her entire existence in Canada and start a new life in America.
(d) The author despised being told by others to break up her life and start again in America.

Head Office: 127, Zone II, MP Nagar, Bhopal |+91-7676564400| https://www.toprankers.com Page 7 of 44
Directions (Q.21-Q.30): Read the passage carefully and answer the questions that follow.

In computer gaming, the term Artificial Intelligence (Al) has always been used with a straight face. The gaming
community got interested in Al in the late 1980s when personal computers started to get more powerful, says
Steven Woodcock, a programmer who has worked in both the defence and games industries. As graphics
improve, he says, a game "needs other discriminators, like whether it plays smart." Game reviews routinely refer
to the quality of the Al —well, what else would you call it? — and some games are renowned for the lifelike
quality of their computer opponents.

Mr. Woodcock says there is now quite a lot of traffic in both directions between Al programmers in the academic
and gaming worlds. Military simulations, he notes, are increasingly based on games, and games programmers

m
are good at finding quick and dirty ways to implement Al techniques that will make computer opponents more
engagingly lifelike. Gaming has also helped to advertise and popularise Al in the form of such impressive games
as The Sims, Black & White and Creatures.

.c o
Another factor that may boost the prospects for Al is the demise of the dotcoms. Investors are now looking for

rs
firms using clever technology. rather than just a clever business model, to differentiate themselves. In particular,
the problem of information overload, exacerbated by the growth of e-mail and the explosion in the number of

k e
web pages, means there are plenty of opportunities for new technologies to help filter and categorise information
— classic Al problems. That may mean that artificial-intelligence startups — thin on the ground since the early

n
1980s — will start to emerge, provided they can harness the technology to do something useful. But if they can,

a
there will be no shortage of buzzwords for the marketing department.

p r
Not everyone is rushing to embrace this once-stigmatized term, however IBM, for example, is working on self-
healing, self-tuning systems that are more resilient to failure and require less human intervention than existing

o
computers. Robert Morris, director of IBM's Almaden Research Centre in Silicon Valley, admits this initiative,
called "autonomic computing", borrows ideas from Al research. But, he says, where Al is about getting

T
computers to solve problems that would be solved in the frontal lobe of the brain, autonomic computing has
more in common with the autonomic nervous system. To some extent, he suggests, the term Al has outgrown its
usefulness. He notes that it was always a broad, fuzzy term, and encompasses some fields whose practitioners
did not regard their work as Al. And while IBM continues to conduct research into artificial intelligence,
Dr.Morris does not link autonomic computing to such work. *This stuff is real," he says.

21. Which of the following about Artificial Intelligence is/are NOT true?
A. The applications of Artificial Intelligence are found in widely
tr-5H7Gdiverse
4D3H1K6Iareas
4A of life.
B. The term Artificial Intelligence was, at one time in the past, a denounced term.
C. Both academic and gaming worlds are discarding the Artificial Intelligence.
D. Firms dealing in Artificial Intelligence use innovative technology to beat competition.
(a) Only C (b) Only D (c) A and C (d) B and D
tr-5L7S 4T3M1M6N4O
22. As inferred from the passage, when Dr.Morris says, "This stuff is real", he means that
(a) the things that were done under the umbrella of Artificial Intelligence had no real value.
(b) what IBM is doing is superior to things that went by the name of Artificial Intelligence.
(c) what was classified as Al was not really intelligent by any stretch of imagination.
(d) the research at IBM is focussed on the autonomic nervous system rather than the frontal lobe of the brain.

Head Office: 127, Zone II, MP Nagar, Bhopal |+91-7676564400| https://www.toprankers.com Page 8 of 44
23. What does the author mean when he says that "In computer gaming the term Artificial Intelligence has always
been used with a straight face"? He means that in computer gaming
(a) the producers have had no hesitation in misleading people with current, catchy names.
(b) Artificial Intelligence has been used freely with little thought of its negative outcomes.
(c) clever technology has been used successfully to lure customers.
(d) the term has been used freely to describe smart applications and lifelike quality.

24. Choose the best alternative to the term ‘straight face’ used in the first line?
(a) Emotions.
(b) Impassiveness.
(c) Responsiveness.

m
(d) Uselessness.

25. What is IBM working on ?


(a) Self-healing.
(b) Self-tuning.

.c o
rs
(c) Less human intervention and resiliency towards failure.
(d) Autonomic computing.

k e
Directions (Q.26-Q.30): Read the passage carefully and answer the questions that follow.

r n
The so-called static gains from trade — the effects of a once-and-for-all shifts in the pattern of production and

a
consumption — are well known. Modern economics also emphasizes the importance of dynamic gains, arising
especially from the economies of scale that freer trade makes possible. The aggregate long-term gain for rich

o p
and poor countries alike is likely to be far bigger than the simple arithmetic would suggest.

Moreover, few displaced rich-country workers are likely to be permanently out of work. Most will move to other

T
jobs. Also, new jobs will be created by the economic opportunities that trade opens up. Overall, trade neither
reduces the number of jobs in the economy nor increases them. In principle, there is no reason to expect
employment or unemployment to be any higher or lower in an open economy as compared to a poor economy.
Still, none of this is to deny that the displaced rich country workers lose out: many, perhaps most, of those who
find alternative work will be paid less than they were before.

In thinking through the economic theory of liberal trade, it is helpful to draw a parallel with technological
progress. Trade, allows a country to shift its pattern of production
tr-5H7Gin4Dsuch
3H1K6Ia4A
way that, after exporting those goods
it does not want and importing those it does, it can consume more without there having been any increase in its
available resources. Advancing technology allows a country to do something very similar: to make more with
less. You can think of trade as a machine (with no running costs or depreciation): goods you can make cheaply
go in at one end, and goods that would cost you a lot more to make, come out at the other. The logic of
protectionism would demand that such a miraculous machine be dismantled and the blueprint destroyed, in order
6N4O
tr-5L7S4T3M1M
to save jobs.

No question, technological progress, just like trade, creates losers as well as winners. The Industrial Revolution
involved hugely painful economic and social dislocations - though nearly everybody would now agree that the
gains in human welfare were worth the cost. Even in far milder periods of economic transformation, such as
today's, new machines and new methods make old skills obsolete. The Luddites understood that, which made
them more coherent on the subject than some of today's skeptics, who oppose integration but not technological
progress. Logically, they should oppose both or neither.

Head Office: 127, Zone II, MP Nagar, Bhopal |+91-7676564400| https://www.toprankers.com Page 9 of 44
Politically, of course, it is essential to keep the two separate. Skeptics can expect to win popular support for the
view that freer trade is harmful, but could never hope to gain broad backing for the idea that, so far as possible,
technological progress should be brought to a halt. Still, it might be better if the scepitcs concentrated not on
attacking trade as such, but on demanding help for the workers who suffer as a result of economic progress,
whether the cause is trade or technology.

26. This passage


(a) analyses the impact of free trade and steps to be taken to mitigate their effect on workers.
(b) argues in favour of free trade while recognizing its positive and negative effects.
(c) compares free trade and technological progress to bring out their similarities.
(d) traces the growth of globalization over the past couple of decades.

m
27. With which of the following would the author of this passage disagree?
A. The long-term benefits of free trade increase exponentially.
B. In general. progress in trade and technology does not hurt workers.
C. Free trade neither reduces the number of jobs nor increases them in an economy.
D. There are no two opinions on the need for technological progress.

.c o
rs
(a) Only B
(b) A and B
(c) C and D
(d) A and D

k e
28.

an
The author draws a comparison between liberal trade and technological progress

r
(a) to show why it is necessary to keep the two apart in considering their impact on society.
(b) to emphasize that the two are inextricably intertwined and their effects cannot be analyzed in isolation.

o p
(c) to exemplify that while the latter is always beneficial, the same is not true about the former.
(d) to argue that both free trade and technology contribute to the worker’s suffering in the name of progress.

29.

T
what does the sentence “technological progress create losers as well as winners” signify?
(a) Technological progress brings in its wake new start-ups and at the same time renders some enterprises
obsolete.
(b) While there are gains in human skills because of technological advancement, there is loss of human welfare.
(c) Equal opportunity of failure and success is similar in trade and technological progress.
(d) While there are gains in human welfare, there is loss of human skills because of certain skils becoming
redundant.
1K6I4A
tr-5H7G4D3H
30. which of the following is correct for this statement “few displaced rich-country workers are likely to be
permanently out of work.?”
(a) Presumption.
(b) Beyond belief.
(c) Improbable. 6N4O
tr-5L7S4T3M1M
(d) Inevitable.

Head Office: 127, Zone II, MP Nagar, Bhopal |+91-7676564400| https://www.toprankers.com Page 10 of 44
SECTION-B : CURRENT AFFAIRS, INCLUDING GENERAL KNOWLEDGE

Directions (Q.31–Q.65): Read the information given below and answer the questions based on it.

Passage (Q.31-Q.35): China is on track to complete construction of its space station by the end of the year,
according to Chinese state media. Chinese authorities said they were preparing a spacecraft that will carry a new
batch of three astronauts to Tiangong, per the China Manned Space Engineering Office (CMSE). The launch is
expected to happen in the coming days. Thereafter, China plans to send two laboratory modules to the space
station in July and October, state-owned CGTN reported on Sunday, quoting information provided by the CMSE.
These modules and astronauts are the final steps in Tiangong's construction, CMSE officials said at a press
conference in April.
"After we complete construction of the space station in orbit at the end of this year, we will start developing use

general, said, according to a transcript from the press conference.

c m
cases and applications for the station that will last over the next ten years," Hao Chun, the agency's director

o
The completion of Tiangong, or "Celestial Palace" as it's called in Chinese, is likely to intensify a brewing space

rs .
rivalry with Beijing and Moscow on one side and Washington on the other. The International Space Station (ISS)
is the only space station that's currently fully operational. The US, Russia, Japan, Europe, and Canada collaborate
on the ISS, according to NASA. China was effectively shut out from the ISS after Congress passed a law in 2011

expected to be the only country to operate its own space station.

k e
banning NASA from collaborating in any way with China. When Tiangong becomes operational, China is

an
(Source: https://www.businessinsider.in/international/news/china-says-its-close-to-completing-its-independent-
space-station-as-beijing-and-moscow-intensify-a-space-rivalry-with-the-us/articleshow/91914429.cms)

r
31.

o p
Which of the following statements is not correct about Tiangong Space Station?
(a) The Tiangong space station is a Chinese space station being built in low Earth orbit between 340 and 450
kilometers above the earth.

T
(b) China is going to operationalize its new Tiangong multi-module space station for at least twenty years.
(c) China launched an unmanned module named "Tianhe", or "Harmony of the Heavens" for its permanent space
station in 2021 that it plans to complete by the end of 2022.
(d) Tianhe core module is the first module to launch the Tiangong space station module.

32. Consider following statements about China Manned Space Programme


I The Chinese government decided to launch a human space programme using a "four-step" method in 1992
which is known as the China Manned Space Program. tr-5H7G4D3H1K6I4A
II It is managed by the China National Space Administration (CNSA).
Which of the above statements is/are not true?
(a) Only I
(b) Only II
(c) Both I and II 6N4O
tr-5L7S4T3M1M
(d) Neither I nor II

33. China is only the third country in history to have put both astronauts into space and to build a space station, after
US and
(a) Russia
(b) Japan
(c) India
(d) Australia

Head Office: 127, Zone II, MP Nagar, Bhopal |+91-7676564400| https://www.toprankers.com Page 11 of 44
34. India’s preliminary plan for the space station is to accommodate astronauts for up to 20 days in space, and the
project will be an extension of the
(a) Gaganyaan mission
(b) Shukryan mission
(c) Antarikshyaan mission
(d) Brahma-I mission

35. Three astronauts entered the orbiting module of the Tiangong Space Station. They were launched into the
designated orbit by the
(a) Wentian-3 spacecraft
(b) Mengtian-11 spacecraft
(c) Chianzou-8 spacecraft
(d) Shenzhou-14 spacecraft

c o m
Passage (Q.36-Q.40): Approximately 3800 youth have registered till 5pm on Friday, the first day of registration,

rs .
for recruitment into the Indian Air Force (IAF) under the new Agnipath scheme, according to an IAF official.
Registration for the IAF is open from June 24 to July 05 while registrations for Army and Navy are set to open
on July 01. According to the schedule given by IAF, the course is set to commence on December 30, 2022. First

k e
of batch of Agniveers are set to arrive at Navy’s training establishment INS Chilka on November 21. In a first,
the Navy will also recruit women Agniveers. The Army plans to induct 40,000 Agniveers in two batches with

n
83 recruitment rallies planned across the country. In the first batch around 25000 Agniveers will be recruited by

a
first half of December and remaining in the second batch by first half of February 2023.

r
On June 14, the Government announced the “Agnipath” scheme for recruitment of soldiers into the armed forces

o p
for four years replacing the current permanent entry scheme and the age bracket for new recruits was fixed at 17
½ to 21 years of age. For this year, the Government has announced a onetime upper age limit relaxation to 23
years. All recruitment in the forces for soldiers, sailors and airmen will now be through this route. Agniveers on

cadre.
T
completion of four years will get an opportunity to join regular cadre and upto 25% would be selected for regular

(Source:https://www.thehindu.com/news/national/iaf-begins-selection-process-under-agnipath-
scheme/article65560586.ece)

36. Which of the following statements is not correct about the Agnipath Scheme?
(a) It allows patriotic and motivated youth to serve in the Armed Forces for a period of four years.
(b) Under this scheme, the youth joining the army will be called
tr-5H7G4D 3H1K6I4A Youth will be able to be recruited
Agniveer.
into the army for a short duration.
(c) Under the new scheme, around 45,000 to 50,000 soldiers will be recruited annually, and most will leave the
service in just four years.
(d) After four years, only 25% of the batch will be recruited back into their respective services, for a period of
6N4O
10 years. tr-5L7S4T3M1M

37. Consider following statements about the profits of Agniveers


I Upon the completion of the 5-years of service, a one-time ‘Seva Nidhi’ package of Rs 11.71 lakhs will be
paid to the Agniveers that will include their accrued interest thereon.
II They will also get a Rs 48 lakh life insurance cover for the five years.
III In case of death, the payout will be over Rs 1 crore, including pay for the unserved tenure.
Which of the above statements is/are not true?
(a) Both I and II (b) Both II and III (c) Both I and III (d) Only III

Head Office: 127, Zone II, MP Nagar, Bhopal |+91-7676564400| https://www.toprankers.com Page 12 of 44
38. Which of the following is the name of India’s own indigenous aircraft?
(a) Rudresh (b) Yashowardhan (c) Soumitra (d) Tejas

39. Who is the current Chief of Army Staff of the Indian Army?
(a) General Rakesh Kumar Singh Bhadauria
(b) General Manoj Pande
(c) General MM Naravane
(d) General Hari Kumar

40. India is the second largest importer in defence procurement in the world after
(a) Russia (b) China (c) Iran (d) Saudi Arabia

m
Passage (Q.41-Q.45): With the installation of ‘Param Pravega’ — claimed to be one of the most powerful

o
supercomputers in the country — at the Indian Institute of Science (IISc), Bengaluru, earlier this month, India

c
has now installed at least three supercomputers in scientific institutions across the country in the last three months

rs .
under the government’s National Supercomputing Mission. So far, ten supercomputers have been installed in
ten institutes with five more in the final stages, according to the department of science.

e
The National Supercomputing Mission is intended to indigenise the development and manufacturing of powerful
computers. This year, nine more supercomputers are likely to be commissioned at IIT Bombay, IIT Madras, IIT

k
Patna, IIT Delhi, IUAC (Inter University Accelerator Centre) Delhi, CDAC (Centre for Development of

n
Advanced Computing) Pune, SNBNCBS (SN Bose National Centre for Basic Sciences) Kolkata, NCRA

a
(National Centre for Radio Astrophysics) Pune, and NIC (National Informatics Centre) Delhi.

p r
Entering the third phase of the mission, design and development of indigenous server nodes, interconnect switch,
storage, and system software stack has been initiated with 85 per cent indigenous manufacturing. The mission

o
was planned in three phases with phase I looking at assembling supercomputers, phase II looking at
manufacturing certain components within the country, and phase III where a supercomputer is designed by India.

41.
T
An indigenously developed server platform called ‘Rudra’ is being tried out in a pilot system, with an
interconnect for internode communication called Trinetra also having been developed.
(Source: https://indianexpress.com/article/india/supercomputers-national-supercomputing-mission-7787052/)

Which of the following statements is not correct about the National Supercomputing Mission?
(a) The National Supercomputing Mission was announced in 2015, with an aim to connect national academic
and R&D institutions with a grid of more than 70 high-performance computing facilities at an estimated cost
1K6I4A
of ₹4,500 crores over the period of Ten years. tr-5H7G4D3H
(b) The mission is being implemented by the Department of Science and Technology (Ministry of Science and
Technology) and Ministry of Electronics and Information Technology (MeitY), through the Centre for
Development of Advanced Computing (C-DAC), Pune and Indian Institute of Science (IISc), Bengaluru.
(c) Under NSM, the long-term plan is to build a strong base of 20,000 skilled persons over the next five years
6N4O
tr-5L7S4T3M1M
who will be equipped to handle the complexities of supercomputers.
(d) None of these

42. What is the name of NSM’s first supercomputer which has been installed in IIT-BHU, Varanasi?
(a) Srijan Vaishnav (b) Brahma Mahesh (c) Param Shivay (d) Abhiraam Shukal

43. Which country has the maximum number of supercomputers?


(a) US (b) Japan (c) France (d) China

Head Office: 127, Zone II, MP Nagar, Bhopal |+91-7676564400| https://www.toprankers.com Page 13 of 44
44. Consider following statements about computing unit of supercomputers
I Petaflops: It is a unit of computing speed equal to one million million (10^12) floating-point operations per
second (FLOPS).
II Teraflops: It is a unit of computing speed equal to one thousand million million (10^15) floating-point
operations per second (FLOPS).
Which of the above statements is/are not true?
(a) Only I (b) Only II (c) Both I and II (d) Neither I nor II

45. The NSM envisaged setting up a network of how many high-performance computing facilities in the country?
(a) 70 (b) 55 (c) 40 (d) 60

m
Passage (Q.46-Q.50): Despite the promise of a worldwide green recovery in the wake of the Covid 19 pandemic,

o
this historic opportunity has been lost, says a report. REN21's Renewables 2022 Global Status Report (GSR

c
2022) sends a clear warning that the global clean energy transition is not happening, making it unlikely that the
world will be able to meet critical climate goals this decade.

rs .
The second half of 2021 saw the beginning of the biggest energy crisis in modern history, exacerbated by the

e
Russian Federation's invasion of Ukraine in early 2022 and unprecedented global commodity shock. "Although
many more governments committed to net zero greenhouse gas emissions in 2021, the reality is that, in response

k
to the energy crisis, most countries have gone back to seeking out new sources of fossil fuels and to burning

n
even more coal, oil and natural gas," said Rana Adib, REN21 Executive Director. The GSR annually takes stock

a
of renewable energy deployment worldwide. The 2022 report, released on Wednesday, is the 17th consecutive

p r
edition and provides proof of what experts have been warning about: the overall share of renewables in the
world's final energy consumption has stagnated-rising only minimally from 10.6 per cent in 2009 to 11.7 per

o
cent in 2019-- and the global shift of the energy system to renewables is not happening.
By the end of the year, gas prices reached around 10 times the 2020 levels in Europe and Asia and tripled in the

T
US, leading to a spike in wholesale electricity prices in major markets by the end of 2021 Russia's invasion of
Ukraine deeply aggravated the unfolding energy crisis, causing an unprecedented commodity shockwave that
weighed heavily on global economic growth, rattling the more than 136 countries that are reliant on fossil fuel
imports.
(Source: https://economictimes.indiatimes.com/industry/renewables/india-ranks-3rd-globally-for-total-
renewable-additions-in-2021-report/articleshow/92246122.cms)
3H 1K6I4A
46. tr-5H7G4D2022
Which of the following statements is not correct about Renewables Global Status Report?
(a) The Renewables 2022 Global Status Report documents the progress made in the renewable energy sector.
(b) It highlights the opportunities afforded by a renewable-based economy and society, including the ability to
achieve more diversified and inclusive energy governance through localised energy generation and value
chains.
tr-5L7S4T 3M1M6N4O
(c) Countries with higher shares of renewables in their total energy consumption enjoy a greater level of energy
independence and security.
(d) None of these

Head Office: 127, Zone II, MP Nagar, Bhopal |+91-7676564400| https://www.toprankers.com Page 14 of 44
47. Consider following statements about Report
I For the first time, GSR 2022 provides a world map of renewable energy shares by country and highlights
progress in some of the leading countries.
II In the lead-up to the United Nations Climate Change Conference (COP26) in November 2021, a record 125
countries pledged to achieve net zero greenhouse gas emissions by 2050.
Which of the above statements is/are not true?
(a) Only I (b) Only II (c) Both I and II (d) Neither I nor II

48. What was the India’s rank in in renewable energy installations in 2021?
(a) 3rd (b) 4th (c) 25th (d) 36th

49. Which of the following mission is wrongly matched with their aims?

energy expansion programme.

c o m
(a) National Solar Mission (NSM): The 100 GW solar ambition at the heart of the world’s largest renewable

manage waste, and create jobs

rs .
(b) National Biofuels Policy and SATAT: Building value chains to reduce fuel imports, increase clean energy,

e
(c) International Solar Alliance (ISA): Harnessing the infinite power for the Sun for sustainable human
development.

communities into the economic mainstream.

an k
(d) National Hydrogen Energy Mission (NHEM): Harnessing the power of water to integrate remote

50.
(a) 35.4 GW (b) 45.3 GW
r
India added 843 MW of hydropower capacity in 2021, raising the total capacity to

p
(c) 51.2 GW (d) 64.3 GW

o
Passage (Q.51-Q.55): There is only one Earth – and it needs to be protected, now.

T
That was the message from scientists, policymakers, and everyday people on Sunday as millions observed World
Environment Day 2022. The event featured several major policy announcements, including a pledge from
Sweden to limit the extraction of coal, oil, and natural gas.
But it also came with a warning that human-induced climate change, nature and biodiversity loss, and pollution
and waste were pushing the planet to the breaking point. UN Secretary-General António Guterres, in his World
Environment Day message, called for a unified global effort to halt environmental degradation.
“We know what to do. And, increasingly, we have the tools to do it,” he said. “But we still lack leadership and
1K6I4A
cooperation. So today, I appeal to leaders in all sectors: leadtr-us5Hout
7G4Dof
3Hthis mess.”
This was echoed by Inger Andersen, Executive Director of the United Nations Environment Programme
(UNEP), speaking at a World Environment Day event in Stockholm, Sweden. “The science has told us we have
to end fossil fuels. We have to restore nature to its full glory. We have to transform our food systems. We have
to make our cities green.”
6N4O
tr-5L7S4T3M1M
https://www.unep.org/news-and-stories/story/world-environment-day-pledges-offer-hope-future

51. Which of the following is the correct expansion of P3 with respect to the mission of the LiFE Movement?
(a) Planned Protection of Planet
(b) People Protecting Planet
(c) Plant Protects Planet
(d) Pro-Planet People

Head Office: 127, Zone II, MP Nagar, Bhopal |+91-7676564400| https://www.toprankers.com Page 15 of 44
52. Which of the following is NOT one of the urgent crises addressed at the Stockholm+50?
(a) Climate change
(b) Protection of highly endangered species
(c) Nature and Biodiversity Loss
(d) Pollution and waste

53. Which of the following is INCORRECT regarding Stockholm+50?


(a) It was co-hosted by Sweden and Finland.
(b) It commemorated the 50th anniversary of the United Nations Conference on the Human Environment.
(c) It reinforced the outcomes of the fifth UN Environment Assembly, which took place in Nairobi, Kenya.
(d) All of the above.

54. Which of the following is the correct full form of LiFE Movement?
(a) Living indigenous for the Environment

c o m
.
(b) Lifestyle ideal for the Environment

rs
(c) Living in For the Environment
(d) Lifestyle for the Environment

55. Which of the following is true regarding India’s LiFE movement?

k e
(a) LiFE Movement will begin the ‘LiFE Global Call for Papers,’ which will solicit ideas and suggestions from

n
academics, universities, and research institutes, among others.

a
(b) The LiFE Movement is a global effort dedicated to improving human sustainability and environmental
protection.

r
(d) All of the above

o p
(c) The LiFE Movement aims to bring positive change in the environment by collective action.

T
Passage (Q.56-Q.60): Posing a challenge for urban planners, UN-Habitat has identified multi-hazard
vulnerabilities, urban sprawl, weak urban mobility and “green-blue disconnect” as the pressing issues for Jaipur
city. Though Rajasthan’s capital has received a high sustainability rating, government agencies have their task
cut out to deal with the city’s expansion.
The global body has based its findings on a sustainable city integrated approach pilot project, as part of which
the “sustainable urban planning and management” component was implemented in collaboration with the Jaipur
Development Authority and Jaipur Greater Municipal Corporation.
Jaipur has received an overall sustainability rating of three on the Urban
7G4D 4A
3H1K6ISustainability Assessment Framework
tr-5H
(USAF) on the basis of data collected for 87 of its 131 parameters. Experts have identified urban sectors where
the city needs to pay maximum attention, and made recommendations to increase its green cover, strengthen
urban biodiversity, and enhance the quality of life for citizens.
UN-Habitat's city coordinator Ankit Gupta told The Hindu that a few critical issues related to the city’s urban
development had been identified to propose strategic interventions 6N4O
tr-5L7S4Tand promote
3M1M sustainable development. The
project has received funding from the GEF-6 to estimate the carbon sequestration potential of Indian cities.
https://www.thehindu.com/news/national/un-habitat-finds-multi-hazard-vulnerabilities-in-jaipur-
city/article65470230.ece

56. Which of the following is the correct expansion of GEF in context of the above passage?
(a) Grant for Environmentally Friendly policy
(b) Global Environment Facility
(c) Global Environment Fund
(d) Grant for Environment Fund

Head Office: 127, Zone II, MP Nagar, Bhopal |+91-7676564400| https://www.toprankers.com Page 16 of 44
57. Which of the following is NOT true regarding UN-Habitat?
(a) It is the United Nations programme for human settlements and sustainable urban development.
(b) UN-Habitat maintains its headquarters at the United Nations Office at Nairobi, Kenya.
(c) It was established in 1978 as an outcome of the United Nations Conference on Human Settlements (Habitat
II) in Istanbul, Turkey, in 1976.
(d) It is a member of the United Nations Development Group.

58. Which of the following is INCORRECT regarding the recommendations of the UN-Habitat for Jaipur?
(a) Increase the green cover, strengthen urban biodiversity, and thereby enhance the quality of life for citizens.
(b) Eco-trails with plantations along natural drainage channels and railway tracks are recommended in the city.
(c) Authorities should measure oxygen produced and carbon dioxide released each day in each urban complex.

59.
(d) None of the above.

Which of the following is TRUE regarding the GEF?

c o m
(a) It is based in Washington DC, United States.

rs .
(b) It addresses four designated focal areas: biodiversity, climate change, land degradation and ozone depletion.

e
(c) India is only a recipient at GEF.
(d) All of the above.

60.

n k
Which of the following have been proposed to be used for mitigating urban floods and efficient use of water
resources in Jaipur?

a
(a) Building artificial ponds
(b) Enlarging the existing bunds in the city

p r
o
(c) Using the dry wells inside the Walled City area.
(d) All of the above

T
Passage (Q.61-Q.65): Today, Prime Minister Narendra Modi inaugurated the biggest Drone Festival in India –
Bharat Drone Mahotsav 2022, at Pragati Maidan in New Delhi. According to a statement issued by the Prime
Minister’s Office (PMO), at the drone festival event, PM Modi will interact with Kisan drone pilots and also
with startups in the drone exhibition centre. Besides, the Prime Minister will also witness open air drone
demonstrations. India’s biggest Drone Festival- Bharat Drone Mahotsav 2022 is a two-day event and it is being
held on 27 May 2022 and 28 May 2022 at Pragati Maidan in the national capital, the PMO statement said.
According to the Prime Minister’s Office, more than 1600 delegates, 1K6I4A
tr-5H7G4D3Hconsisting of government officials, armed
forces, central armed police forces, Public Sector Units (PSUs), foreign diplomats, private companies as well as
drone startups, etc., will participate in the Bharat Drone Mahotsav 2022. The PMO mentioned that more than 70
exhibitors will display various use cases of drones at the Drone Festival. The Bharat Drone Mahotsav 2022 will
also witness a virtual award of drone pilot certificates, panel discussions, product launches, display of a ‘Made
6N4O
in India’ Drone Taxi prototype, flying demonstrations, amongtr-others, the
5L7S4T3M 1MPrime Minister’s Office further added.
https://www.financialexpress.com/infrastructure/bharat-drone-mahotsav-2022-pm-modi-inaugurates-indias-
biggest-drone-festival-at-pragati-maidan/2539550/

61. Which of the following is INCORRECT with regard to Drone Rules, 2021?
(a) Permission is now required for operating drones in green zones.
(b) It created several new permissions and approvals to regulate the use of drones in Indian airspace.
(c) Remote pilot license is necessary for non-commercial use of micro and nano drones as well.
(d) All of the above

Head Office: 127, Zone II, MP Nagar, Bhopal |+91-7676564400| https://www.toprankers.com Page 17 of 44
62. Which of the following is the correct full form of DrAAS in the context of the above passage?
(a) Drone Applications And Services
(b) Drone As A Service
(c) Drone And its Applicable Services
(d) Drone As A Saviour

63. Which of the following is/are NOT an application of Kisan Drones?


(a) Crop assessment
(b) Digitisation of land records
(c) Spraying of insecticides and nutrients
(d) None of the above

64. Which of the following is TRUE regarding the future of Drones in India?
(a) It will help in furthering the vision of saturation and in ensuring last-mile delivery.
(b) Foreign ownership of companies operating drones will soon be permitted in India.

c o m
(d) All of the above

rs .
(c) Payloads up to 100kg will soon be allowed so that drones can be used as unmanned flying taxis.

65. SVAMITVA Scheme of the Union Government is for which of the following purposes?
(a) To provide a comprehensive pollution assessment.

k e
(b) A symmetric weapon against terrorist attacks.

an
(c) Integrated property validation solution for rural India.
(d) Monitoring of forests and wildlife.

r
o p
T
1K6I4A
tr-5H7G4D3H

6N4O
tr-5L7S4T3M1M

Head Office: 127, Zone II, MP Nagar, Bhopal |+91-7676564400| https://www.toprankers.com Page 18 of 44
SECTION - C: LEGAL REASONING

Directions (Q.66-Q.105): Read the comprehension and answer the questions:

Passage (Q.66-Q.70): Negligence, in law, is the failure to meet a standard of behaviour established to protect
society against unreasonable risk. Negligence is the cornerstone of tort liability and a key factor in most personal
injury and property damage trials. The doctrine of negligence does not require the elimination of all risk from a
persons’ conduct—only all unreasonable risk, which is measured by the seriousness of possible consequences.
The standard of behaviour is external. Generally, the law examines only conduct, not the excitability, ignorance,
or stupidity that may cause it. The courts determine what the hypothetical “reasonable person” would have done
in the situation. Such standards also demand a degree of foresight in anticipating the negligence of others—
especially of special groups such as children. The reasonable-person test presumes certain knowledge—e.g., that

m
fire burns, water may cause drowning, and cars may skid on wet pavement. Community custom will influence
such presumptions, such as the practice of driving on a certain side of the road even on private roads, a situation

. o
in which laws do not apply. Emergencies, however, can soften the application of such standards. Allowances

c
may be made for physical (but not mental) disabilities, such as blindness, but the law demands that disabled
persons avoid needlessly placing themselves in situations in which their disability may cause harm. Other than

rs
distinguishing between children and adults, the doctrine of negligence does not usually consider factors of age
or experience. Ordinarily, the plaintiff in a negligence suit must prove the defendant’s negligence by a

k e
preponderance of the evidence, which may be circumstantial so long as it is not too speculative. In some
situations, once the plaintiff has established an apparent connection between his injury and the defendant’s

n
apparent negligence, the latter must disprove that connection. This is the doctrine of res ipsa loquitur. Generally,

a
the damages recoverable for negligence are monetary compensation for injuries or losses that are deemed to have

r
flowed “naturally and proximately” from the negligent act.
https://www.britannica.com/topic/negligence

66.

o p
Which of the following statement cannot be concluded from the given passage?
(a) The foundation of tort liability is negligence, which is a major factor in most personal injury and property
damage cases.

have caused it. T


(b) In general, negligence law looks at conduct rather than the excitement, carelessness, or idiocy that may

(c) In most negligence cases, the plaintiff must prove the defendant's negligence by clear and convincing
evidence, which can be circumstantial if it isn't overly subjective.
(d) The defendant must prove the plaintiff's apparent relationship between his injury and the defendant's
apparent fault once the plaintiff has disproven it.
1K6I4A
tr-5H7G4D3H
th
67. Two teams were playing cricket. During the 5 over of the match, the bowler bowled an in-swinger which the
batsman tried to hit for a boundary towards the “long on”. Fortunately, there was a fielder who ran towards the
ball, stopped it and with the intention to run out the batsman threw the ball towards stumps. Due to bad aim the
ball hit the umpire, who was very far from the stumps, in his face and he had to be hospitalised immediately.
Determine the liability of the fielder as per the given passage. 6N4O
tr-5L7S4T3M1M
(a) The fielder is not liable because he did not have the intention to hit the umpire, it happened by mistake.
(b) The fielder is liable because he was ought to have a good aim and not injure anyone with his throw.
(c) The fielder is not liable because his actions do not come under the scope of ‘negligence’.
(d) The fielder is liable because he clearly fails the reasonable person test.

Head Office: 127, Zone II, MP Nagar, Bhopal |+91-7676564400| https://www.toprankers.com Page 19 of 44
68. Suppose in the previous question, the umpire was just beside the stumps and not very far, when the ball thrown
by the fielder hit him in the face. With the slight change in the facts, determine if your answer to the previous
question would change or not?
(a) No, the fielder is liable because he does not satisfy the reasonable person test.
(b) Yes, the fielder is liable because he ought to have a good aim and not injure anyone with his throw.
(c) No, he is not liable because the ingredients of negligence are not satisfied.
(d) Yes, he is not liable because his action, in this case, will not come under the scope of negligence.

69. Examine the given statement and choose the option which can be directly inferred from it. “Ordinarily, the
plaintiff in a negligence suit must prove the defendant’s negligence by a preponderance of the evidence, which
may be circumstantial so long as it is not too speculative. In some situations, once the plaintiff has established

m
an apparent connection between his injury and the defendant’s apparent negligence, the latter must disprove that
connection.”

defendant, then the tort of negligence will not be applicable.

. o
(a) If there is no direct link between the actions of the plaintiff and the subsequent harm caused to the

c
(b) The burden to prove/disprove a connection between plaintiff’s injury and defendant’s negligence can shift

rs
to the other party.
(c) The court provides an opportunity to the defendant to disprove the charges on him via any means he deems
fit.

k e
(d) The evidence provided by the victim has to be contextual to the suit raised, else the case will be dismissed.

70.

an
In which of the following cases the doctrine of res ispa loquitur will not be applicable?

r
(a) X rides his cab in heavy rain, hits a pole, and injures his passenger.
(b) X, a bartender makes ice cubes of tap water, which when consumed by people in their drink, makes them
sick.

o p
(c) X, a sweeper is moping the floor and a few people fall while walking on the wet floor, injuring themselves.
(d) X is trying to fly a kite, but when he loses control of the same, he tries to land the kite on his neighbour’s

T
terrace, which hits the neighbour in his eye, who was sleeping there.

Passage (Q.71-Q.75): Section 2 (d) of the Contact Act 1872 defines consideration as “When at the desire of the
promissory, the promise or any other person has done or abstained from doing or does or abstains from doing or
promises to do or abstain from doing something such act or abstinence or promise is called a consideration for
the promise.” Consideration is the foundation of every contract. The law insists on the existence of a lawful
consideration if a promise is to be enforced as creating legal obligations. A promise without consideration is null
and void. There are three types of Consideration tr-5H7G4D3H
1K6I4A
• Executory: Executory consideration is a promise given for a promise.
• Executed: Executed consideration is an act in return for a promise
• Past consideration: Past consideration which as general rule is not sufficient to make the promise binding. In
such a case the promisor may by his promise recognize a moral obligation (which is not consideration), but
he is not obtaining anything in exchange for his promise (as he already has it before the promise is made).
6N4O
tr-5L7S4T3M1M
In order to constitute legal consideration, the act or abstinence forming the consideration for the promise must
be done at the desire or request of the promisor. Thus, acts done or services rendered voluntarily, or at the desire
of third party, will not amount to valid consideration so as to support a contract. The second essential of valid
consideration, as contained in the definition of consideration in Section 2(d), is that consideration need not move
from the promisee alone but may proceed from a third person. It may move from the promisee or from any other
person. This means that even a stranger to the consideration can sue on a contract. This is sometimes called as
‘Doctrine of Constructive Consideration’. The words, “has done or abstained from doing; or does or abstains
from doing; or promises to do or to abstain from doing,” used in the definition means that a consideration may
consist of a past, present or a future act or abstinence. Consideration may consist of an act or abstinence.

Head Office: 127, Zone II, MP Nagar, Bhopal |+91-7676564400| https://www.toprankers.com Page 20 of 44
71. Anand offers a reward for the return of lost property, provided the finder of goods brings the lost property in
exact condition and without any damage. Radhe finds the property in perfect condition and brings it to Anand.
After due verification, Anand pays him the reward. Going by the facts of the case, what is the nature/type of
consideration involved?
(a) Executed Consideration
(b) Executory consideration
(c) Past consideration
(d) Can’t be determined as facts are incomplete

72. Subodh is a 19-year-old college student. His father is extremely disturbed by his lack of seriousness towards the
career. In order to secure his future, the father enters into a contract with the career counsellor to pay them

m
₹15,000 per month in case they are successful in convincing Subodh to look for better career opportunities. The
career counsellor successfully convinces Subodh and now seeks INR 15,000 from him. Subodh refuses to pay

him.

. o
any such fee. The career counsellor sues Subodh for not paying the promised consideration for services given to

c
(a) Subodh is not bound to pay anything as no act was performed by the career counsellor at his desire.

rs
(b) Subodh is not bound to pay anything as a stranger to a contract cannot be sued.
(c) Subodh is bound to pay since the career counsellor rendered services only on the promise of INR 15,000 per
month
(d) Subodh is bound to pay as a stranger to consideration can sue and be sued.

k e
73.

an
Salil is a famous singer of Bollywood who is being signed by a lot of film makers for singing songs in their

r
movies. Sudeep is a big producer in Bollywood and has signed Salil for singing in his movie. Sudeep offered
Salil INR 1.5 Lakhs on a condition that Salil shall not sing the said song in any other movie, show or anywhere

o p
else where live singing is required. Salil agrees to the said condition. However, later it comes to Subodh’s
knowledge that Salil has sung that song in a friend’s party. Subodh refuses to pay INR 1.5 Lakhs to Salil for
failing to fulfill his condition. Salil sues him saying that there was no valid consideration from Salil. Decide if

T
there was a valid consideration from Salil
(a) There is no valid consideration since Salil did not give anything in return to Sudeep
(b) There is a valid consideration since at the desire of Sudeep, Salil promised to not sing the said song at any
event
(c) There is a valid consideration since Salil has sung the said song for Sudeep and singing the song is enough
consideration
(d) This is not a question of consideration as Salil merely sung at a friend’s party which caused no loss to Sudeep
1K6I4A
tr-5H7G4D3H
th
74. Suresh is a 5 year law student who believes in social service. He once saw an old lady fighting against illegal
eviction from her house by her son. Suresh offered to help her. After the case was won and the old lady won
compensation of INR 15 Lakhs. Out of the said compensation, Suresh took 2 Lakhs as a reimbursement of
expenses incurred in the case. The old lady sought her money back by arguing that there was no promise to make
any payment. Suresh contended that since he offered to help, she is liable to pay. Decide which option amongst
6N4O
tr-5L7S4T3M1M
the following is appropriate
(a) There is no binding contract due to lack of consideration and hence, Suresh cannot claim any money
(b) There is a binding contract since Suresh offered to help her. Since there was offer and acceptance, there is a
contract. Hence, he can claim money
(c) Suresh can claim money as it is a past consideration for the services rendered by him
(d) Suresh cannot claim money as no promise to pay was made by the old lady at the time of entering into the
contract.

Head Office: 127, Zone II, MP Nagar, Bhopal |+91-7676564400| https://www.toprankers.com Page 21 of 44
75. Ridhima was a college student. Ridhima’s parents lost their job during COVID in 2020. Due to the same,
Ridhima was facing severe financial constraints and was in dire need of money. She received an offer for help
from Stephen, his classmate, on a condition that Ridhima shall help Stephen in passing an exam by sharing her
answers in the middle of the exam. Ridhima agreed to the same and during exams, helped Stephen in passing.
After the exams, Ridhima asked Stephen for money, however he refused. Can Ridhima sue him for money?
(a) Ridhima cannot sue him as there was no formal contract
(b) Ridhima cannot sue him as there was no valid contract
(c) Ridhima can sue him as there was executed consideration from her end
(d) Ridhima can sue him even though there was no valid consideration since no person can take undue advantage
of other.
Passage (Q.76-Q.80): One of the most essential elements of a valid contract is the competence of the parties to make

m
a contract. Section 11 of the Indian Contract Act, 1872, defines the capacity to contract of a person to be dependent

o
on three aspects; attaining the age of majority, being of sound mind, and not disqualified from entering into a contract

c
by any law that he is subject to. According to Section 11, “Every person is competent to contract who is of the age of

rs .
majority according to the law to which he is subject, and who is of sound mind and is not disqualified from contracting
by any law to which he is subject.”. The age of majority in India is defined as 18 years. Since any person less than 18
years of age does not have the capacity to contract, any agreement made with a minor is void ab-initio (from the

k e
beginning). A void agreement cannot be ratified. While a minor cannot enter a contract, he can be the beneficiary of
one. Even if a minor falsely represents himself as a major and takes a loan or enters into a contract, he can plead
minority. The rule of estoppel cannot be applied against a minor. He can plea his minority in defence. Under certain

n
circumstances, a guardian of a minor can enter into a valid contract on behalf of the minor. Such a contract, which the

a
guardian enters into, for the benefit of the minor, can also be enforced by the minor.

r
According to section 12 of The Indian Contract Act,1872, A person is said to be of sound mind for the purposes

o p
of making a contract, if, at the time when he makes it, he is capable of understanding it and of forming a rational
judgment as to its effect upon his interests. The crucial point, therefore, is to find out whether he is entering into
the contract after he has understood it and has decided to enter into that contract after forming a rational

76.
1872/] T
judgement in regard to his interest
[source- Our Legal World, https://www.ourlegalworld.com/capacity-to-contract-under-indian-contract-act-

Tushar was born on 12th June 2000. Since the age of 5, Tushar had deep interest in cars and their parts. On 10 th
June 2018, he saw an offer to buy a sports car at 50% discount. He was intrigued by it. He convinced his parents
to let him buy the car. He took money from his father and went to purchase the car at 50% discount. However,
he promised to give the money 10 days later but he failed to do so. Car seller is looking to sue him but he pleads
6I4A
5H7G4D3H1K
incompetency to contract on grounds of minority. Can he dotr-so?
(a) He cannot plead minority as the contract was entered into for the benefit of the minor since the car was
available at 50% cost
(b) He cannot plead minority as he took the consent of his father/guardian before entering into the contract
(c) He can plead minority as he was a minor at the time of entering into the contract
(d) He can plead minority as he would have attained the age of 187Syears
tr-5L 4T3M1Mon the date the payment became due.
6N4O

77. Shweta, 17 years of age, has just completed her school and also cracked CLAT UG exam. She has joined a law
college; however, the fee of college is extremely high. Her parents decide to take an education loan in her name
for the purpose of her college and hostel fee. Will this loan agreement be valid in light of the passage above?
(a) The contract is not valid since Shweta was a minor at the time of making the loan agreement
(b) The contract is valid since the agreement is made for her benefit i.e., for the purpose of her educational needs
(c) The contract is not valid as her parents cannot take any loan in her name since she is a minor and loan is not
made for her benefit
(d) The contract is valid due to authority vested in parents to take such life decisions for their child.

Head Office: 127, Zone II, MP Nagar, Bhopal |+91-7676564400| https://www.toprankers.com Page 22 of 44
78. Sonal had entered into a contract for the sale of her favourite study table at the age of 17.5 years. However, after
entering into a contract, the buyer came to know that Sonal was only a minor at the time of entering into the
contract. On turning 18, she asked the buyer for continuation of the old contract for the sale of her study table.
The buyer agreed. Decide the validity of the latter contract
(a) The contract is not valid as the agreement made at the time of minority cannot be ratified, at the age of
attaining majority, even by mutual consent of parties
(b) The contract is valid as the agreement made at the time of minority can be ratified with mutual consent of
parties
(c) The contract is valid as the continuation of an old contract is not ratification.
(d) The contract is valid as Sonal had falsely represented herself as a minor at the time of first contract.

79. Based on the passage above, decide which of the following agreements shall be valid in the eyes of law

m
(a) An agreement with a guardian, on behalf of the minor, for the purpose of paying school fee.
(b) An agreement with a guardian, on behalf of the minor, for the purpose of selling minor’s house.

.c o
(c) An agreement with a guardian, on behalf of the minor, for the purpose of purchasing a watch
(d) An agreement with a guardian, on behalf of the minor, for the purpose of paying sports club fee

rs
80. Sukhdev was a brilliant investment banker and due to his expertise and skills, he accumulated a net worth of 200
Crore. During his old days, when he turned above 75 years, he hired a trustee for all his properties and securities

k e
and also hired a caretaker. Sukhdev fell extremely ill in his 80s and barely was able to get out of bed or do any
personal chores by himself. His caretaker and trustee did everything for him. They both made him sign a will to

n
transfer all his wealth to the trustee and caretaker, while he was on his death bed. The legal representatives of

a
Sukhdev challenged the validity of the will. Decide, in light of the passage.

r
(a) The contract of will is not valid as it suffers from coercion and undue influence
(b) The contract of will is void since Sukhdev, being a party to the contract, suffered from unsoundness of mind

it

o p
(c) The contract of will is void since Sukdhev, at the time of making a contract, did not understand the nature of

(d) The contract of will is valid since Sukhdev did not suffer from lunacy or other types of unsoundness of mind.

T
Passage (Q.81-Q.85): Neither the substantive law nor procedural law defines the term child witness. However,
section 118 of Evidence Act contemplates that "All persons shall be competent to testify unless the court consider
that they are prevented from understanding question put to them or from giving rational answers to those
questions, by tender years, extreme old age, disease whether of body or mind, or any other cause of the same
kind."
The Evidence Act does not prescribe any particular age as a determinative factor to render a witness to be
competent one. Herein, the concept of Voir dire test derivedtr-from
5H7G4DAnglo- Norman phrase which refers to "Oath
3H1K6I4A
to tell the truth”, assumes importance. The test can be called as precursor test which is supposed to be conducted
by the trial court to determine the maturity and capability of the child. In that test, the judge must ascertain the
competency of the child by asking some questions unrelated to the case on hand, before testifying the child
witness as a part of trial proceedings.
However, subsequently it was held by the Supreme Court that mere non-compliance
6N4O
of Voir Dire Test would
5L7S4T3M1M
not ipso facto lead to the whole sale rejection of the evidencetr-of child witness when it inspires confidence and
trust worthy of the court.
The tender years of the child, coupled with other circumstances appearing in the case, such as his demeanour,
unlikelihood of tutoring and so forth, may render corroboration unnecessary but that is a question of fact in every
case. The only rule of law is that this rule of prudence must be present to the mind of the Judge as the case may
be understood and appreciated by him. There is no rule of practice that corroboration for the testimony of child
witness is sine qua non for every case to sustain conviction.
[Extracted with revisions https://blog.ipleaders.in/witness-under-the-evidence-act-1872/]

Head Office: 127, Zone II, MP Nagar, Bhopal |+91-7676564400| https://www.toprankers.com Page 23 of 44
81. Bittu was an 11-year-old child who worked at a tea stall in Jamnagar. One day, he saw a man driving away at a
very high speed in a BMW car. Later, it was found that the car had been stolen and the person driving the car
was actually the thief. Bittu was presented in court as a child witness. The Judge allowed him to speak and Bittu
narrated exactly what he saw without adding or subtracting anything. His statements remained consistent during
the cross-examination as well. The judge was satisfied with Bittu's statements and the thief was convicted. Is the
conviction sustainable if Bittu was the sole witness?
(a) Yes, because the judge failed to apply the rule of prudence in understanding and appreciating the facts of the
case
(b) Yes, because the judge applied his mind and was satisfied with Bittu’s statements because of his consistency
and unlikelihood of tutoring
(c) No, because the testimony of a child witness like Bittu cannot be the sole basis of conviction
(d) No, because the judge failed to apply the rule of prudence in understanding and appreciating the facts of the

m
case

82.

.c o
In the previous fact situation, Jian, the convict, challenged his conviction. He argued that during the trial, the
judge failed to ask any questions to Bittu to ascertain his competency and maturity. In light of the same, the
judge could not have placed sole reliance on Bittu's testimony to convict Jian. Is Jian correct in his assertions?

rs
(a) Yes, because the judge must ascertain the competency of the child by asking some questions unrelated to the
case on hand, before testifying the child witness as a part of trial proceedings

it inspires confidence of the court

k e
(b) Yes, because mere non-compliance of Voir Dire Test would not lead to the rejection of the evidence when

n
(c) No, because merely not asking questions to ascertain Bittu’s competency would not lead to the rejection of

a
the evidence when it inspires confidence of the court

r
(d) No, because non-compliance of Voir Dire Test leads to the ipso facto rejection of the evidence of the child
witness

83.

o p
Subsequently, Jian's arguments were rejected by the Court. He then asserted that the judge did not corroborate
Bittu's testimony. Based on the same, he submitted that his conviction could not be considered sustainable. If

T
Jian's assertion is true and Bittu's testimony is uncorroborated, can Jian's conviction be sustained?
(a) Yes, because Jian cannot be convicted on the sole uncorroborated testimony of a child witness due to the
immaturity and tender age of the witness
(b) Yes, because there is no necessity that the testimony of a child witness must be corroborated in order to
sustain conviction
(c) No, because as per rule of practice the testimony of a child witness must be corroborated in order to sustain
conviction
(d) No, because Jian cannot be convicted on the sole uncorroborated testimony
tr-5H7G4D3H
1K6I4A of a child witness due to the
immaturity and tender age of the witness

84. Shinchan's parents were always fighting. Often, their arguments became so heated that they both ended up using
physical force on each other, even in front of Shinchan. Eventually, his parents filed for divorce. While hearing
their plea, the family court called Shinchan to the witness box to hear1Mthe real story through his words, even
6N4O
5L7S4T3M
though he was just 5 years old. Shinchan withstood the crosstr-examination and answered many vital questions
with the nod of head in an intelligent and coherent manner. As a result, the divorce was granted because his
testimony inspired confidence in the Court. Is this in consonance with the principles outlined in the passage?
(a) No, because Shinchan is merely 5-years-old and such a young child cannot be expected to testify in Court
(b) No, because there is no rule of practice that corroboration for the testimony of child witness is sin qua non
for every case to sustain conviction.
(c) Yes, because only rule of law is that the rule of prudence must be present to the mind of the Judge as the
case may be understood and appreciated by him
(d) Yes, because the tender age of Shinchan coupled with other circumstances appearing in the case, such as
demeanour, likelihood of tutoring and so forth, may render corroboration necessary
Head Office: 127, Zone II, MP Nagar, Bhopal |+91-7676564400| https://www.toprankers.com Page 24 of 44
85. Nobita was a child witness in a robbery case. While giving his testimony, he hesitated several times and it
appeared as if he was trying to remember something. His statements did not appear as spontaneous. At one point,
he said that "I stated that in the court which was what Nana asked me to." Nobita's testimony was rejected on the
ground of tutoring. Based on your reading of the passage, is this correct?
(a) Yes, because factors like Nobita’s demeanour and likelihood of tutoring did not inspire confidence and trust
of the Court
(b) Yes, because Nobita’s testimony inspired confidence and trust of the Court and could have been relied upon
(c) No, because Nobita’s testimony inspired confidence and trust of the Court and could have been relied upon
(d) No, because factors like Nobita’s demeanour and likelihood of tutoring did not inspire confidence and trust
of the Court

m
Passage (Q.86-Q.90): A conspiracy is an unlawful combination of two or more persons to do that, which is
contrary to law, or to do that which is harmful towards another person, or to carry out an object not in itself

lawful act by unlawful means.

. o
unlawful by unlawful means. It may consist in the agreement of two or more to do an unlawful act or to do a

c
In medieval England there was a system of state's regulation of labor and prices. The workmen and their leaders

rs
were punished for criminal conspiracy if they made demands for higher wages. There were laws against
combinations. When persons combined to pervert the ends of justice, they were liable for tort of conspiracy if
damage was caused.

k e
Later on, statutes gave protection to laborers by the Conspiracy and Protection of Property Act of 1875 and the

n
Trade Disputes Act of 1906. The tort of conspiracy is committed when two or more persons combine in such a

a
way that their conduct amounts to criminal conspiracy which is a punishable wrong. Special damage must be

r
proved to make it actionable. It is also necessary that the defendant must have done something in pursuance of
conspiracy. The action cannot lie for acts, which are not unlawful. Section 43 of the Code defines 'illegal' as

o p
"applicable to everything which is an offence or which is prohibited by law; or which furnishes ground for a
civil action." Hence persons can be made liable for tort of conspiracy if they combine to commit assault, libel,
trespass, etc.

T
In India, after the adoption of the Trade Unions Act, 1926, trade union workers and leaders are immune from
civil and criminal proceedings when there is a trade dispute. But by the Industrial Disputes Act, 1947, where in
certain cases the trade union and its officials are not allowed to act, they can be held liable for civil action of
conspiracy.
(Source:https://www.nascollege.org/e%20cotent%201020/dr%20chandan%20uadhyay/Law%20of%20Torts.p
df)

86. Assertion: No agreement except an agreement to commit an tr- unlawful


5H7G4D3Hact
1K6Ishall
4A amount to a criminal conspiracy.
Reason: Law making conspiracy a crime is designed to curb immoderate power to do mischief which is gained
by a combination of the illegal means.
(a) Both A and R is true but R is correct explanation of A.
(b) Both A and R is true and R is not correct explanation of A.
(c) A is true but R is false. 6N4O
tr-5L7S4T3M1M
(d) A is false but R is true.

87. Choose a statement that does not illustrate a false assertion around the concept of criminal conspiracy:
(a) The gist of the offence is an agreement to break the law.
(b) The parties will be guilty of criminal conspiracy, even though the illegal act agreed to be done has not been
done.
(c) Offence of criminal conspiracy is exception to the general law where intent alone does not constitute crime
(d) It would not be enough for the offence of conspiracy when some of the accused merely entertained a wish,
howsoever, horrendous it may be, that offence be committed.

Head Office: 127, Zone II, MP Nagar, Bhopal |+91-7676564400| https://www.toprankers.com Page 25 of 44
88. Raju was planning to surrender his licensed gun at a local police station, as arms licensees have been advised to
deposit their weapons before upcoming polls. He noticed a crowd gathering near the red hand bus stop. To find
out what was going on, he discovered that a number of opposition party supporters were calling their followers
and indirectly persuading the crowd to protest the government's plan to build new malls across the city by
chopping down thousands of trees, causing flora and wildlife to suffer. Raju is convinced by the leader’s speech.
The cops arrived and detained everyone in the throng on grounds of criminal conspiracy. Decide
(a) Raju will be held liable for being a part of a criminal conspiracy.
(b) Raju will not be held liable for any offence.
(c) Raju will be held liable to be part of conspiracy as he is convinced of the objective.
(d) Raju will not be held liable for being a part of criminal conspiracy.

89. Tomal Pandey is the fashion icon who unabashedly owns her unconventional styles and inspires through the

m
process. She is popular for creating content for digital platforms like YouTube and Instagram and is also regarded
as a well-known influencer by fashion and make-up brands. She received a lot of hate comments including being

. o
called immoral, characterless because she showed too much skin. One Tara and Sara, who share an Instagram

c
account, commented on one of her photos, saying she has no regard for Indian culture and that her account should
be removed. They also said that she is a loser since she failed her B.com test and abandoned her studies to pursue

rs
a career as a fashion influencer. Can Sara and Tara be held liable for conspiracy?
(a) Yes, both will be held liable as they have casted aspersions on her character on a social media platform.

express their views.

k e
(b) No, they will not be held guilty for conspiracy because the comment section is used by many people to

n
(c) Yes, both will be held liable for committing the tort of conspiracy.

a
(d) Yes, they will be found accountable for criminal conspiracy since a claim for criminal conspiracy is

r
actionable in the event of special damage.

90.

o p
Suryansh and Madhu is a newlywed couple. Madhu, who came from a middle-class background, was often
taunted by her in-laws and husband. This did not end there; her mother-in-law turned the mockery into an assault
when she threatened her for not bringing enough gifts from her home in the wedding. She also tricked her son

Decide T
into believing that they don't complement one other. As a result, her husband began assaulting her for the same
reason. Madhu filed a tort of conspiracy case against her husband and mother-in-law for committing assault.

(a) It is not a case of conspiracy as both the accused acted separately.


(b) It is a case of conspiracy as both the accused acted jointly to assault Madhu.
(c) It is a case of criminal conspiracy; not a tort of conspiracy.
(d) It is a case of conspiracy as it involved two or more persons combining in such a way that their conduct
amounts to criminal conspiracy which is a punishable wrong.tr-5H7G4D3H
1K6I4A

Passage (Q.91-Q.95): A "bailment" is the delivery of things by one person to another for a certain purpose, with
the agreement that after the purpose is fulfilled, the commodities will be returned or disposed of according to the
instructions of the person who delivered them. Here the "bailor" is the one who delivers the goods, and the
"bailee" is the one to whom the goods are being delivered. Personal property 6N4O
involves only movable property
tr-5L7S4T3M1M
and excludes immovable. Chapter nine of the Indian Contract Act, 1872 discusses the whole concept of Bailment.
Bailee has been entrusted with several duties. The foremost is that he has to take care of the goods bailed
reasonably. Section 151 of the Indian Contract Act states, "In all cases of bailment, the bailee is bound to take
as much care of the goods bailed to him as a man of ordinary prudence would, under similar circumstances,
take of his own goods of the same bulk, quantity and value as the goods bailed." The degree of care that should
be exercised should be comparable to the level of care that an ordinary person would exercise in the same
circumstances with his own goods of the same bulk, quantity, and value as the items bailed. The instant section
does not provide any exception with the general provisions, so it hardly matters what type of the goods are bailed,
i.e., the degree of care for all kinds of Bailment. This stance has not always been consistent. Bailee's liability
was initially absolute under English law. This law put the bailee in a problematic situation because there would
Head Office: 127, Zone II, MP Nagar, Bhopal |+91-7676564400| https://www.toprankers.com Page 26 of 44
be no recourse even if a reasonable degree of care were exercised to safeguard the items. When it comes to
Bailment, Section 151 establishes a common standard of care that must be followed in all circumstances. It also
provides that if the bailee has exercised reasonable care, the bailee will not be held liable for any loss, damage,
or degradation of the commodities bailed.
(Source: https://articles.manupatra.com/article-details/A-bailee-s-duty-to-take-care-of-the-goods-bailed-as-
like-a-prudent-man)

91. Certain goods were consigned for the respondents by the third party from Calcutta to Delhi by Railway.
However, the receipts were endorsed in favor of respondents. During the transit from Calcutta to Delhi, some
items were stolen, resulting in not being delivered to the respondents. The respondents filed an action against
Railways. In the trial court proceedings, it was unveiled that the Goods were adequately sealed and loaded when
the train left, but after two hours, it was found that the seal of the wagon was broken. There was an incident of

m
theft. The question before the Court was the act of the railway authorities can be considered negligent and failed
to take reasonable care?

keeping a watch on his belongings, especially when the train comes to a stop.

. o
(a) Yes, as it goes without saying that a regular person traveling by train would be particularly vigilant in

c
(b) No, as it is not the responsibility of the railway to take care of the consignment but of the delivery agent.

rs
(c) Yes, as a bailee, railway is required to take as much care of the goods bailed to it as it would care for its
own goods.

would care for his goods.

k e
(d) Yes, as a bailee, railway is required to take as much care of the goods bailed to it as a prudent person

92.

an
Continuing with the similar set of facts as stated above, suppose the railway administration requires that its

r
employees, particularly those who have been specially deputed to make sure that no harm is committed to the
goods, get down from their wagons during stoppages and keep a more vigilant eye on the wagons in the train.

for losses?

o p
The railway employees are not diligent. Consider the given facts and decide whether or not the bailee is liable

(a) Railway as a bailee will be held liable for the losses.

93.
T
(b) Employees of the railway will be liable for the losses
(c) Railway will not be held liable for losses since it took reasonable care.
(d) Railway and its employees will be held liable for losses.

Bhide as the secretary of the Gokuldham co-operative society was being given cash collected by the society for
repairing works. Bhide as a usual practice kept the cash in society’s locker room. He locked the cash and kept
the key in its designated place where it was usually kept in everyday practice. Even then, the cash was stolen. In
the given situation, which of the following statements is true?
tr-5H7G4D3H
1K6I4A
(a) Bhide will be held liable for stolen cash as Bhide was negligent although he followed a usual practice, that
practice was full of insecurity and risk.
(b) Bhide will not be held liable for stolen cash as Bhide followed the usual practice.
(c) Bhide will be held liable for not taking reasonable care of the goods bailed as he should have not followed
the usual practice. 6N4O
tr-5L7S4T3M1M
(d) Bhide will not be held liable as he exercised reasonable care against the goods bailed.

Head Office: 127, Zone II, MP Nagar, Bhopal |+91-7676564400| https://www.toprankers.com Page 27 of 44
94. Consider the following example and select the statement in which the bailee is liable for failing to take due care
of goods bailed:
I. At the time of the admission Sheela gave some jewellery to the hospital officials for safe custody.
Jewellery was kept by Hospital official in the locker room, from where it was stolen.
II. A lends a horse to B for his own riding only. B allows C, a member of his family, to ride the horse. C rides
with care, but the horse accidentally falls and is injured.
III. A person deposits a fresh and crisp Rs. 500 note in his savings account. At the time of withdrawal, the
Bank offers him an old looking and much used Rs. 500 note.
(a) II and III (b) I and II (c) III (d) II

95. Dunzo is an Indian company that delivers groceries and essentials, fruits and vegetables, meat, pet supplies, food,

m
and medicines in major cities. It also has a separate service to pick up and deliver packages within the same city.
Raju has an order in the post office that is yet to be delivered to him. When he contacted and enquired, he learned

. o
that he must personally travel to the post office and pick up his order. Raju then hired a delivery agent from

c
Dunzo to go to the post office, pick up the order, and bring it to him. His mother sent him homemade culinary
sauces from the United States. It was raining heavily that day, and when Raju received the delivery, he discovered

rs
that all of the contents had been spoiled. Consider the facts and make a decision on who will be held responsible
for damages.
(a) Dunzo will be held liable for the damages occurred to the Raju
(b) Delivery agent will be held liable for the damages occurred to Raju

k e
n
(c) Post office will be held liable for the damaged goods delivered to Raju.

a
(d) The liability in regards with the damages to goods cannot be ascertained.

p r
Passage (Q.96-Q.100): During the nationwide lockdown during the first wave of the covid-19 pandemic in
India, the CM of Delhi in a televised press conference promised to pay rent for those who could not pay.

o
However, no action was taken in this regard. In July 2021, in a landmark judgement the Delhi HC held that the
oral promise made by the CM to be legally enforceable citing the doctrine of promissory estoppel. It held that

T
promises made by government functionaries, no less than the head of government in this case, are enforceable
in law even if they’re mere oral promises and ordered the Delhi government to pay the rent.
The High Trees House case is an English decision which explained and affirmed this principle of PE (Promissory
Estoppel) in contract law and common law. It is based on the idea that one cannot make a promise that another
relies upon, even if not formally in writing, and then renege on it. When this representation relates to future
course of action, or a ‘promise’ about one’s future conduct, it becomes ‘promissory estoppel’.
It should however be noted that PE is not an absolute doctrine to bind the government. If the government can
show an overriding public interest reason for which it cannottr-5Habide
7G4D3Hby
1K6Iits
4A promise, then public interest must
prevail and the government is not bound.
(Source: https://lawandotherthings.com/2022/02/explainer-delhi-hc-held-that-a-public-promise-by-a-cm-is-
enforceable-in-court-to-what-extent-and-what-does-it-mean/)

96. Consider the following statements, choose the one which does not illustrate an oddity with passage’s text:
6N4O
5L7S4T3M1M
I. Any promise made by the government contrary to any lawtr-would not be enforceable.
II. Estoppel cannot be used to override any statute or law.
III. To have binding effect a promise needs to be based on a valid representation or conduct.
(a) II and III (b) I (c) III (d) All of the above

Head Office: 127, Zone II, MP Nagar, Bhopal |+91-7676564400| https://www.toprankers.com Page 28 of 44
97. CM had promised an ex-gratia award of Rs. 1 Crore to a police constable who died from Covid as a frontline
warrior during the first national lockdown, and tweeted about the same. The deceased constable’s widow has
now filed a petition to enforce the promise of the ex-gratia award. Can she do so?
(a) No, as not every representation can be claimed as promissory estoppel.
(b) No, as the government is estopped by the CM’s statement in the press conference.
(c) Statement made by the CM constituted an unequivocal promise by the government to pay, and this promise
is bound by promissory estoppel.
(d) No, as the CM only promised a favor moreover the wife did not act upon the representation made.

98. The DTC had passed a resolution to sell allotted houses to retired employees. However, a later report found that
there was already a shortage of houses to allot to existing employees and the cost of replacing the houses to be

m
sold was too high for the DTC to afford. Hence, DTC denied selling the house as promised earlier to the retired
employees. Choose the correct statement, in light of the facts and information stated in the passage.

(b) Promissory estoppel is not an absolute doctrine to bind the government.


(c) The government is bound by the representations made by its functionaries.

. o
(a) There is a sufficient public interest reason for DTC to not be bound by its earlier resolution.

c
rs
(d) The representation was made in writing i.e., in form of a resolution, hence DTC has to comply with the same.

99.

e
The chief secretary of UP had assured the petitioner that the tax exemptions would be granted, on the basis of

k
which the petitioner proceeded to set up an industrial project. The UP government refused to grant the said tax

n
exemptions that it had notified, relying on which the petitioner had secured a business loan. Decide.

a
(a) The government will be estopped from granting the announced exemption to the petitioner.

r
(b) The government is bound because it has a duty to act fairly and reasonably.
(c) The government cannot be held absolutely bound under the doctrine of promissory estoppel.

o p
(d) The government is bound under the doctrine of promissory estoppel to implement the exemptions.

100. The CM of Punjab announced in a public rally for milk producers that the government had abolished a tax on

T
milk. This was reiterated by the finance minister in the budget session of the state assembly, and subsequently
in many government advertisements and a cabinet announcement said that a notification would be issued soon.
Relying on all this, the petitioner, Jestle India did not file tax for that year to which no protest was raised.
However, a year later a demand was made to pay the past year’s tax. Determine if the doctrine of promissory
estoppel will be applicable in this case or not.
(a) Oral representations by government officials have been held to create justiciable promises in certain
exceptional cases.
(b) Yes, as the representation and acts by government functionaries give
tr-5H7G4D3H
1K6Ithe
4A reasonable impression that the tax
was abolished.
(c) Yes, and the petitioner should be exempted from paying the tax.
(d) Yes, as there came no protest against the petitioner for the year it failed to file tax.

Passage (Q.101-Q.105): The killing of gangster Vikas Dubey by the Uttar Pradesh Police has put the spotlight
6N4O
tr-5L7S4T3M1M
back on encounters or executive killings. The legality of police action in such circumstances has been debated
for long, and a legal framework was put in place with the intention of establishing accountability. The Supreme
Court and the National Human Rights Commission (NHRC) have framed guidelines that are to be followed in
cases of custodial deaths.
In 1993, the Commission had issued general guidelines that every case of custodial death must be intimated to
it within 24 hours. Further, the post mortem reports, inquest requests, and other related documentation was to be
sent to the human rights watchdog to ascertain its reliability within two months of the incident.
If a death is prima facie found to be a case of death that took place unlawfully, the Commission would grant
compensation to the victim's kin and penalize the errant state and its officials.

Head Office: 127, Zone II, MP Nagar, Bhopal |+91-7676564400| https://www.toprankers.com Page 29 of 44
In 1997, then NRC chairperson M.N. Venkatachalaiah, in a letter addressed to chief ministers of all states, had
emphasised that a policeman, if found responsible for a custodial death, would have the same defences available
in law that are available to the common man, and would have no special protection.
Essentially, this meant that for every case of custodial death, the concerned officers would be on trial, and their
actions would not constitute an offence in only two circumstances: a) if they have killed the individual in order
to protect themselves and, b) if use of force extending to death is necessary for making an arrest.
An FIR is registered under Section 302 of the Indian Penal Code which penalises culpable homicide. The Indian
Evidence Act puts the burden of proof on the defence - the police in this case - to prove that the offence was not
committed.
(Source: https://indianexpress.com/article/explained/vikas-dubey-kanpur-encounter-law-nhrc-guidelines-
explained-6501890/)

m
101. The killing of Lakhan in an encounter with the army in Baksa, Assam, on February 11, 2012, was reported to
the National Human Rights Commission. The Enquiry Magistrate had noticed various inconsistencies in Army

. o
personnel's accounts and concluded that the fire was driven by fear of attack. Whereas no firing took place from

c
the side of the deceased while he was trying to run away, the Magistrate found. Is the encounter valid?
(a) Yes, as it was driven by fear of attack and the deceased was fleeing away.

rs
(b) No, as in the opinion of the magistrate the army personnel gave contradictory statements.
(c) Yes, as use of force extending to death is necessary for making an arrest.
(d) Encounter was not genuine, as the act was not done in self defence.

k e
n
102. The Delhi Police killed two terrorists in an encounter in the capital, claiming that the action was taken in self-

a
defence. The fatal gunshot wound suffered by Inspector MC Sharma and the serious injury sustained by Head

r
Constable Balwant Singh during the event corroborated the police version that the residents of the flat raided by
the police fired first. The matter has been referred to the National Human Rights Commission. Choose a

o p
statement that provides the correct ruling in the present case.
(a) The police party’s conduct is not legitimate but not unlawful either as they acted in self defence.
(b) The encounter was legitimate because the alleged terrorists resorted to fire, causing significant injuries.

T
(c) The burden of proof lies on the police in this case - to prove that no offence was committed by its personnel.
(d) Police party's conduct in self-defense was legitimate because the alleged terrorists fired first.

103. The National Human Rights Commission (NHRC) has taken up the case of eight SIMI activists who were being
held in the Bhopal prison and died in an encounter with the Bhopal police after attempting to flee. The NHRC
investigation corroborated the SIMI activists' charges of torture in the Bhopal jail. Select the appropriate
statement.
(a) The panel will hold the state and its officials accountabletr-for
5H7Gthe activists'
4D3H 1K6I4A deaths.
(b) The defence must prove that the alleged offence has not been committed.
(c) If the captives attempt to flee, the police officers have the right to shoot them.
(d) Not every encounter is considered a crime.

104. Which of the following statements are true in regards with the information presented in the passage?
6N4O
5L7S4T3M1M
(a) Vikas Dubey encounter with the Uttar Pradesh Police hastr-brought to the fore a host of questions about the
legal validity of such encounters.
(b) The hold a policeman guilty of custodial death his guilt must be established beyond reasonable doubt.
(c) While extrajudicial killings are not a new phenomenon in India, and have been witnessed across the country
over decades. Police officers responsible for alleged extrajudicial killings are not held to account.
(d) Encounter reports are important because they bring out the “extent of the cover up” in cases like this, and
highlight the injustices being done.

Head Office: 127, Zone II, MP Nagar, Bhopal |+91-7676564400| https://www.toprankers.com Page 30 of 44
105. Banrakas, an infamous robber, robbed people by pretending to be a victim of robbery and seeking their
assistance. Once inside their vehicle he would threaten the occupants and rob them of their valuables. One day
he sought assistance of Suman, a doctor. Suman helped him and offered to take him to the nearest hospital. Later,
Suman’s car was discovered along with Suman’s mutilated body hidden in the trunk. This led to an outrage in
social media and news channels. Everyone started questioning the law-and-order situation in the state. Police
nabbed the culprit within a few hours. Banrakas admitted sexually assaulting the doctor and killing her. Many
people surrounded the police personnel and demanded that Banrakas be punished there and then as they had little
faith in judicial process. Later, the dead body of Banrakas was discovered with four bullet wounds 50 metres
away from the spot where he was arrested. Police claimed that they fired because Banrakas was trying to flee.
Decide.
(a) Justice has been delivered in this case. Police team should be rewarded for quick arrest of the culprit.
(b) Police team is justified in shooting Banrakas as he was trying to flee.

m
(c) Police team isn’t justified in shooting Banrakas as the same wasn’t necessary.
(d) Police team is justified in their action as Banrakas confessed his crimes and ignoring the wishes of the mob
gathered on the spot would have led to a tricky law-and-order situation.

.c o
k e rs
r an
o p
T
1K6I4A
tr-5H7G4D3H

6N4O
tr-5L7S4T3M1M

Head Office: 127, Zone II, MP Nagar, Bhopal |+91-7676564400| https://www.toprankers.com Page 31 of 44
SECTION - D: LOGICAL REASONING

Directions (Q.106-Q.135): Read the passage given below and answer the questions that follow-

Directions (Q.106-Q.110): read the following passage and in the light of same, answer the questions that follow
Deforestation has historically been the price of development, but the world is now going through a forest
transition; since 2015, there has been net global reforestation. The pace and quality of this transition is mixed.
In the world’s remaining high conservation-value forests, deforestation rates are high and poverty persists but
development opportunities are within sight. These forests are mainly located in the tropical developing world
and have growing human populations. Because as forest dependent people are increasingly involved in cash
economies, they use their forests to participate in markets. This will inevitably lead to changes in forests.
Conservationists typically respond to this in one of two ways. They either tackle the threat head on and try to

m
counter it or they hand over forest management to local people. But neither threat-based conservation nor local
management has proven successful for preserving forests. Tropical forests are still subject to high deforestation

. o
rates in less developed countries and conservationists lament the ongoing decline in biodiversity and ecosystems.

c
To local people, tackling threats to forests is seen as opposing development and ergo it will continue to fail.
Opposing new roads in areas where people are deprived of development opportunities, for instance, is clearly

rs
not a viable way forward. But the alternative strategy of handing management over to local people in the hope
that they will protect biodiversity is also not pragmatic. If the only options on offer are forest protection or

k e
development at their expense (plantations, for instance, or mines and agriculture) then most people will,
naturally, choose the latter. It’s mindful that local peoples’ needs and aspirations change with time. People living

n
near forests will say their preferred future includes the continued existence of forests and therefore, biodiversity

a
and intact ecosystems. The challenge is to achieve this alongside improvements in livelihoods.

r
[Source: https://www.business-standard.com ]

o p
106. On the basis of information provided above, why would the local people fail in preserving forests?
(a) Because the inherent nature of people is self-serving and everyone wants to live comfortably
(b) Because for the local people, intervention is bad and ought to be stopped

forests
(d) Both (a) & (b) T
(c) Because the local people hate to share their resources with others which results in decommercialization of

107. Which of the following statements is in line with the author’s line of thoughts?
(a) Deforestation and reforestation are happening all around but the movement is sporadic and in need of an
analysis
(b) Human population and forests can be separated but the tr-local
5H7Gpopulation
4D3H1K6I4A need more convincing
(c) Developing countries have more deforested land than developed countries
(d) Local population can make forest conversation if they are paid monthly wages for the same

108. Which of the following, if true, would seriously weaken the author’s argument?
(a) Many conservationists are actually pro-local people and believe in1M leaving
6N4O
the forests in their hands
5L7S4T3M
(b) The government has enough resources to form a group oftr-forest rangers that may protect the affected areas
(c) Big corporations are the major threat to deforestation in less developed countries
(d) None of the above

109. The author’s statement that “To local people, tackling threats to forests is seen as opposing development
and ergo it will continue to fail”-
(a) Forms premise of the above passage
(b) Forms conclusion of the above passage
(c) Forms assumption of the author to the above passage
(d) None of the above
Head Office: 127, Zone II, MP Nagar, Bhopal |+91-7676564400| https://www.toprankers.com Page 32 of 44
110. What could be a suitable theme for the passage?
(a) Forests are the lungs of the Earth
(b) Once green is today’s obscene
(c) Conserving forests: beyond ideas of local integration
(d) Local communities and their life

Directions (Q.111-Q.115): Read the passage and answer the questions that follow-
1. Irrespective of whether one is a renunciate or a householder, the words of wisdom contained in the
Bhagawad Gita teach us the method of leading a balanced life – a life that attaches equal importance to the
spiritual aspect of life and attainment of proficiency in worldly affairs. The Gita urges repeatedly that one
should live in the world like a lotus leaf, which remains pure and untainted despite being surrounded by filthy

m
water. The Gita opens with a vivid description of men and things on the eve of the great battle of Kurukshetra.
At the eleventh hour, Arjun’s mind is tormented at the sight of his venerable preceptors and grandsons

o
who are present in the battlefield. The very thought of fratricidal carnage and bloodshed that would ensue

c
causes untold agony to his delicate mind – so much so that he puts aside his bow and arrow and is in no mental

.
condition to fight. The Gita elaborates on whether action or renunciation of action is better and concludes that

rs
action is a better option. The discourse on Samkhya Yoga lays stress on renunciation of action. This is because
every deed, irrespective of whether it is good or bad, brings about its natural effect and binds one to the world,

k e
posing an impediment to liberation. It is, therefore, argued that one must renounce all action and become a
sannyasin. At the same time, the teacher of the Gita asserts that abstention from action is just not possible. The

n
Gita thus is a mandate for action; it teaches that perfection can be attained even while doing one’s duties. Gandhi

a
ji in his commentary on the Gita opined that it is not desire that vitiates the action but selfish desire, that is

r
the desire for fruit; and if an end is put to this selfish desire___________. The abandonment of the desire for
fruit encompasses both ‘sannyas’ and ‘tyag’, both having the etymological meaning ‘give up’. In this context,

o p
the person who completely abstains from performing action is not a sannyasi but only an idler or hypocrite.
Krishna says in the Gita: “He who performs all actions in a spirit of surrender to the Eternal Being, abandoning
attachment, is as unaffected by sin as a lotus leaf by filthy water.” The Gita thus advocates detachment from

T
desires and not cessation from work.
SOURCE: http://auraofshirdisai.org/sai-baba-as-gitacharya/ - An excerpt from the Gita.

111. An Analogy has been drawn in the passage. Identify it.


(a) The analogy between lotus leaf and unaffection.
(b) The analogy between detachment and renunciation.
(c) The analogy between agony and bloodshed.
(d) The analogy between spiritual life and worldly affairs. tr-5H7G4D3H
1K6I4A

112. Which of the following strengthens the passage?


(a) The renunciation of all desires and the work is propagated by Lord Krishna to Arjuna.
(b) The abandonment of the selfish desires is the only way a man can reach his highest potential
(c) Performing actions for the welfare and a spirit of surrender can even1Mwash
6N4O
away the sins.
tr-5L7S4T3M
(d) Both a and b strengthens the passage.

113. What is the logical continuation of the line above?


(a) The actions which you perform will not be binding.
(b) The desires will wash away, thereby, liberating yourself.
(c)The longing for the fruit of action will also wash away.
(d) The person abandoning it will experience bliss.

Head Office: 127, Zone II, MP Nagar, Bhopal |+91-7676564400| https://www.toprankers.com Page 33 of 44
114. Which of these are the logical Inferences which can be drawn?
(a) The Gita opens with a crude description of men and things on the night of great battle of Kurukshetra.
(b) Arjuna was fearful of the bloodshed that would occur after the war, hence he completely surrendered himself
to the Lord.
(c) Both a) and b) are the logical inferences.
(d) Neither a) nor b) are the logical inferences.

115. Which of the following weakens the passage, the most?


(a) Samkhya Yoga teaches renunciation of action, to renounce all actions which do not bind us further.
(b) The Gita encapsulates the teachings of Lord Krishna to Arjuna
(c) Attachment to the fruits of action leads to liberation while detachment leads to misery.

m
(d) Detachment to the fruits of action leads to liberation while attachment leads to misery.

o
Directions (Q.116-Q.120): read the following passage and in the light of same, answer the questions that follow

c
Disasters occur when people are affected by natural or technical hazards – when lives are lost or property is

.
destroyed. Research conducted in Sri Lanka suggests that while heavy rainfall is the trigger for the flooding, the

rs
root causes of the disaster are social, in particular widespread poverty, conflict-induced migration and
problematic land-use practices. These characteristics are not homogeneous, meaning different places and people

k e
are affected differently. The social characteristics of communities are extremely important for hazard managers
because they increase peoples’ vulnerability to hazards. Understandably, economically disadvantaged

n
communities exposed to hazards have to date received the most attention from Disaster Risk Reduction (DRR)

a
specialists. This is because hazards tend to harm predominantly those social groups that were already

r
disadvantaged before a disaster. Large focus has been placed on “underdeveloped” or “developing” nations,
where the economic disadvantage factors are particularly obvious. However, examples, from affluent and less

o p
affluent countries, suggest the need to consider economic vulnerability in more geographically and
demographically nuanced ways when implementing DRR activities. On the one hand, poorer communities might
bring alternative capabilities to DRR that are non-financial. On the other hand, ignoring existing economic

T
disadvantage within affluent contexts risks significant loss of life and property, and forgoes the opportunity to
improve the circumstances of the affected sub-populations. Across all stages of the disaster cycle – preparedness,
response, and recovery – knowledge about the nature and location of economically vulnerable groups is critical
for effective DRR. Before an event, knowing which groups have low levels of preparedness is essential for
planning tailored risk communication and support initiatives. During a disaster, information on vulnerable
groups can help to increase the effectiveness of response measures, for example, by establishing priorities during
evacuations.
[Source: https://theconversation.com/understanding-the-root-causes-of-natural-disasters-80017
tr-5H7G4D3H
1K6I4A ]

116. Which of the following, if true, weakens the authors claim?


(a) Sri Lanka is heavily prone towards natural disasters
(b) We do not have the appropriate technology to predict most natural disasters
(c) In majority of the affluent nations, the rich usually inhabit areas that are more prone to destruction during
6N4O
tr-5L7S4T3M1M
disasters than any other areas
(d) Poorer nations are severely affected by natural disasters that take a toll on their economy

117. All of the following can be inferred except?


(a) DRR specialists need to create uniform plans that need to be enforced across all nations irrespective of their
affluence
(b) Previous surveys and information can help better prepare from natural disasters
(c) The trigger and root cause of natural disasters may be different
(d) The poorer regions have it worse when it comes to natural disasters

Head Office: 127, Zone II, MP Nagar, Bhopal |+91-7676564400| https://www.toprankers.com Page 34 of 44
118. Which of the following statements will the author deem the truest?
(a) Mexico had an earthquake which was easily managed considering the financial aid they received from
international bodies.
(b) Cancun had various problems with floods as their government refused to rectify the sewage systems
(c) Parts of Laos still haven’t recovered from the thunderstorm and majority of the people in penury are still
homeless
(d) Kolkata has installed a new software that helps in calculating the impact of natural disasters 24 hours in
advance

119. Which of the following conclusions can be properly drawn from the statements above?
(a) The DRR specialists need to create disaster management dependent on the financial situation of a
country and identify different groups of people that may be affected

m
(b) Even developed nations have poor populations that may be similarly affected as the people from
developing nations
(c) Both (A) and (B)
(d) Neither (A) nor (B)

.c o
rs
120. The author’s statement that “These characteristics are not homogeneous, meaning different places and
people are affected differently”-
(a) Forms premise of the above passage
(b) Forms conclusion of the above passage

k e
n
(c) Forms assumption of the author to the above passage

a
(d) None of the above

p r
Directions (Q.121-Q.125): Read the passage given below and answer the questions that follow-
A mature legal system goes a step further and endeavours to provide not merely a remedy for every right

o
infringed, but also an adequate remedy. It was in this process of a search for effective remedial action that
Specific Relief emanated from the Equity Courts in England. The principles laboriously built up by successive

T
Chancellors of England in this branch of law have been copiously borrowed by the Indian Courts and have
served to enrich Indian Law. This fertilization of Indian Law by the equity Jurisprudence of England produced
in due course the Specific Relief Act of 1877.
General principles of interpretation go like this - the will of the legislature is the supreme law of the land and
demands perfect obedience. “Judicial power is never exercised to give effect to the will of the Judges: always to
give effect to the will of the Legislature; or in other words, to the will of the law”. The language of a statute
should be interpreted in its plain grammatical sense. The interpretation should advance the object of the Act.
Restrictive provisions should not be liberally construed or extended
tr-5H7G4D3H through
1K6I4A the medium of interpretation. The
natural meaning of the words should be accepted though it may produce unfortunate results. Construction should
be avoided which would reduce the legislation to futility.
The object of the Specific Relief Act is confined to; that class of remedies which a suit seeks to obtain and a
court of justice seeks to give him the very relief to which he is entitled. The Law of Specific Relief seeks to
implement the idea of Bentham, who said: “The law ought to assure1M me everything which is mine, without
6N4O
tr-5L7S4T3M
forcing me to accept equivalents, although I have no particular objection to them”. The Law of specific
performance essentially belongs to the law of procedure. It supplements various substantive laws such as the
law of Contracts, the Transfer of Property Act, and the Sale of Goods Act.
That is why the opening section of chapter II of the Act specifically says that the defendant can plead by way of
defence on any ground which is available to him under the law relating to contracts or civil procedure code. The
Honorable Justice Subba Rao observed that in England, the relief of specific performance pertains to the domain
of equity: in India, to that statutory law. In England, mere delay or laches is a ground for refusing the specific
performance, but in India, mere delay without such conduct on the part of the plaintiff would because prejudice
to the defendant, does not empower the court to refuse the relief.
Source- https://districts.ecourts.gov.in/sites/default/files/jcjpalasaspdt31-08-2019.pdf
Head Office: 127, Zone II, MP Nagar, Bhopal |+91-7676564400| https://www.toprankers.com Page 35 of 44
121. Which of one of the following statements resonates with the domain of equity as promulgated by Justice
Subba Rao regarding specific relief in contracts?
(a) The remedy provided is that suitor is granted the very same thing to which he is entitled, rather than
monetary compensation in lieu thereof.
(b) The court's power to grant specific performance is discretionary and based on justice, equity and good
conscience in England.
(c) It is based on the premise that there might be situations wherein a grant of the compensation would not
afford adequate relief and only specific performance of the contract would render justice and provide
adequate relief.
(d) All of these

m
122. Which one of the following is the basis of the Specific Relief Act as can be assumed from the text?
(a) Law of equity (b) Common Law (c) Roman Law (d) British law

.c o
123. In which way the Law of Specific Relief seeks to implement the idea of Bentham as stated in the passage?
1. The Act comes into play when actual damage for non-performance of a contractual obligation cannot be

rs
measured or the monetary compensation would not suffice.
2. The Act provides for a Specific performance of a contract in absence of monetary relief, which makes it an
alternative remedy.
(a) Only 1 (b) Only 2 (c) 1 and 2

k e
(d) Neither 1 nor 2

discussed EXCEPT –
(a) Civil cases of buyer and seller

r an
124. The Specific relief act supplements the law of contracts or transfer of property acts in all the cases

(b) Breach of contract in tenant and owner


(c) Criminal Cases about parties

o p
(d) Illegal possession of land by the trespasser

125. What can be concluded from the statement given below?

T
“In England mere delay or laches is a ground for refusing the specific performance, but in India, mere delay
without such conduct on the part of the plaintiff as would because prejudice to the defendant, does not
empower the court to refuse the relief.”
1. Indian law has reduced the discretionary jurisdiction of the Court in granting specific performance of a
contract.
2. Our law seeks to mandate every court to grant specific performance of the contract as a general rule rather
than making it an exception.
3. Due to the discretionary power, the court would opt fortr-granting
5H7G4D3Hdamages
1K6I4A instead of Specific performance
but the amendment seeks to change that power by making Specific performance a mandate.
(a) Only 1 (b) 1 and 3 (c) 2 and 3 (d) All of these

Directions (Q.126-Q.130): Read the passage given below and answer the questions that follow-
The devaluation of legislative democracy is evident in the poor attendance 6N4O
of public representatives in both
tr-5L7S4T3M1M
houses and other important committees. The demand for sending Bills to a Parliamentary Standing Committee
is constant but when it comes to attendance in its meetings, statistics show a very dismal picture. The latest
statistics provided by Rajya Sabha sources showed that around 100 out of 248 Lok Sabha and Rajya Sabha MPs
in eight panels under the Rajya Sabha have not attended two or more meetings since its reconstitution in
September this year.
The abject state of affairs is not limited to standing committees only. Election season has impacted attendance
negatively. Punjab, UP and Uttarakhand saw only 20-30 days of assembly in the last six months. This is below
the national average of 65 which is much poorer than the national chambers of other democracies like the USA,
Japan and Canada

Head Office: 127, Zone II, MP Nagar, Bhopal |+91-7676564400| https://www.toprankers.com Page 36 of 44
Poor attendance directly indicates that fewer questions were asked and a lack of interest of representatives to
discuss the important issues of society. Fewer sittings raise questions on the governance. Recently, the Chief
Justice of the Supreme court noted in a case that poorly drafted legislation is the result of the apathy of
legislatures in discussing them properly. The other outcome is that the executive, whether at state levels or the
Centre, feels increasingly unconstrained by the legislature. Only 13% of bills in the current, 17th Lok Sabha
have been referred to standing committees, down from 27% in the previous one, and sharply down from over
60% during the 15th Lok Sabha. It is no accident that higher courts are the institutions that act as the only
effective checks on executive overreach.
It is suggested from various corners of concern that the rule of a minimum of 100 sittings shall be put in place
to rein the lawmakers. Since culpability on this is cross-party, the senior leadership of all major parties should
agree at least on this one thing – if they are as committed to India’s democratic system as they all say they are.
Many senior leaders, across parties, increasingly see governance as almost solely an exercise in executive power,

m
and Houses as most venues for political agendas. If legislatures continue to remain as unimportant in governance
as they are now, the decline in the quality of democracy, which is different from just winning elections, may
become irreversible. That’s a truly troubling thought.

.c o
https://timesofindia.indiatimes.com/blogs/toi-editorials/house-that-shockingly-low-legislature-sittings-cant-go-
on-leaders-must-understand-the-quality-of-democracy-is-at-stake/

126. What is the prime concern of the author in the passage?


(a) Devaluation of democracy.
(b) Poor attendance of public representatives.

k ers
n
(c) Apathy of representatives for the problems of the society.

a
(d) Fewer sittings of houses in comparison with other countries.

views?

p r
127. Which of the following statement is a logical explanation of the Chief Justice of the Supreme Court’

(d) None of these


To
(a) His concern is that level of democracy is high in other democratic countries.
(b) Executive branch does not consult the judiciary before drafting the legislation.
(c) The MLAs/MPs consider party duties in elections more important than constitutional duties.

128. What is the logic behind the demand for minimum sittings rules?
(a) The author has imitated the foreign countries in deciding this number.
(b) He left other days for work in constituencies and personal requirements.
(c) It will ensure that proper and sufficient debate on important topics happens.
(d) It will make legislatures responsible for their constitutional duties.
tr-5H7G 4D3H1K6I4A

129. What can be concluded about the value of parliament for politicians?
(a) They do not respect its rules.
(b) They understand and appreciate the role of parliament in democracy.
(c) They underestimate the role of debates in the parliament.
4T3M1M6N4O
(d) Politicians use its platform for their political agendas. tr-5L7S

130. What is not discussed as a negative effect of the fewer assemblies in the house of parliaments?
(a) Lack of progressive debates on issues of importance.
(b) Poorly drafted legislation.
(c) Ineffectiveness of Parliamentary committees.
(d) Unconstrained executive

Head Office: 127, Zone II, MP Nagar, Bhopal |+91-7676564400| https://www.toprankers.com Page 37 of 44
Directions (Q.131-Q.135): Read the passage given below and answer the questions that follow-
A friend would phone me at the stroke of midnight and wish me my birthday, but I would have no recollection
of our zero-hour exchange when I woke up. She would tease me that I was such a baby, and yet, would go
through the midnight ritual each year, knowing fully well that I would be
'dead to the world' at that unearthly hour. But with the technology boom and with advancing years, alas, a
night of undisturbed eight hours of sleep is becoming a pipe dream. Digital platforms and e -devices have
contributed to sleep becoming a casualty, across age groups. Whether this is due to addiction to e -games,
OTT platforms, obsession with breaking news and/or being active on social media, the fact is, most of us are
living sleep-deprived lives. This is tiring; it can negatively impact mental and physical health and in extreme
cases, could even cause death, say experts.
Magazine’s podcast, associate professor of neurobiology at Harvard Medical School, Dragana Rogulja says

m
that she and her team found that when animals and insects are deprived of sleep for several days, they simply

o
crash and soon die. This can happen with human beings as well. Human biology is similar in many ways to

c
the biology of other species, and all species need sufficient sleep to be able to function, build immunity and

rs .
survive well. The 86-year-old XIV Dalai Lama says he sleeps for at least nine hours daily. He goes to bed
early and rises before dawn, meditating and chanting in the quiet of those tranquil early hours. He says
maintaining a consistent sleep schedule is a major factor contributing to good health and well -being, and

k e
longevity. For a healthy immune system, you need a fair supply of microbiome, the good bacteria in our
bodies directly linked to our circadian rhythms. If babies don’t sleep enough, their gut bacteria will not be
able to digest nutrients from what they eat. In a vicious cycle, poor gut health impacts sleep negatively and

r an
lead to sleep loss. When babies sleep, they heal and grow.
Vishnu the Sustainer is often depicted as Ananthasayana, lying on the cosmic serpent that coils itself into a bed,

p
positioning its many hoods canopy. Vishnu goes into a deep slumber on this bed floating on the milky ocean,
his head resting on his palm. During Chairmans, the four-month-long rainy season, Vishnu remains in deep

o
sleep. This is his way of unwinding after working hard, and it is believed that when he wakes up refreshed,

T
creation gets rejuvenated and energized.
131. Which of the following statement, if true, weakens the claims of the author in the passage?
(a) Too much sleep regularly can increase the risk of heart diseases.
(b) It is said that people who sleep lose most of their opportunities in life.
(c) Successful and fit people sleep only for a couple of hours in a day.
(d) Sleeping means wastage of the limited time available in one’s life.
132. What is the most logical inference based on the passage? 1K6I4A
tr-5H7G4D3H
(a) Oversleeping is better than sleep deprivation for good health.
(b) Dalai Lama is free from any type of mental or body disease.
(c) Social media and mobiles keep us unnecessarily occupied to disturb our sleep.
(d) God Vishnu works in summer and rests in the rainy season for four months.
133. According to the passage, which one of the following is not atr-benefit 6N4O
of1Msleep?
5L7S4T3M
(a) Health and longevity. (b) Healing and growth.
(c) Good digestion. (d) Healthy relationships.
134. Which of the following part can be deleted from the passage without losing its effectiveness and critical
message?
(a) The episode of the author and her friend.
(b) The illustration of the Dalai Lama
(c) The illustration of Lord Vishnu
(d) Dragana Rogulja’s Harvard experiment

Head Office: 127, Zone II, MP Nagar, Bhopal |+91-7676564400| https://www.toprankers.com Page 38 of 44
135. Which of the following summarises the passage most logically and rationally?
(a) Sleeping for 8-9 hours has a good impact on mental and physical health, it reflects in the life of great
personalities.
(b) Sleeping changes, the circadian rhythms of the body to increase the level of microbes in human bodies to
bring health and vitality to life.
(c) Sleeping for a specific time without disturbance leads to increased immunity, health and rejuvenation as
found in research and vetted by the Dalai Lama and the mythological stories.
(d) Sleeping is healthy for human beings as it gives rest from the stress and the daily routine by increasing
immunity.

c o m
rs .
k e
r an
o p
T
1K6I4A
tr-5H7G4D3H

6N4O
tr-5L7S4T3M1M

Head Office: 127, Zone II, MP Nagar, Bhopal |+91-7676564400| https://www.toprankers.com Page 39 of 44
SECTION - E: QUANTITATIVE TECHNIQUES

Directions (Q.136-Q.140): Study the following information carefully and answer the question given below it.
Out of the 15,000 candidates eligible for an Officer’s post in a Public Sector Bank, 450 candidates have prior
experience of working in Public Sector Banks in rural areas only. 25% of the total number of candidates have
prior experience of working in Public Sector Banks in urban areas only. 12% of the total number of candidates
have prior experience of working in Private Sector Banks in urban areas only. 2% of the total number of
candidates have prior Experience of working in Private Sector banks in rural areas only. 3,600 candidates have
worked in both Public and Private Sector Banks in urban areas only. 600 candidates have worked in both Public
and private Sector Banks in rural areas only. The remaining candidates have no prior experience of working in
the Banking industry.

m
136. Candidates having prior experience of working in rural areas is what percent of candidates having experience of
working in urban areas?(round off to 2 decimal places)
(a) 13.75% (b) 13.25% (c) 15.25% (d) 14.75%

.c
137. What percent of total number of candidates have prior experience of working in Public Sector Banks ?o
rs
(a) 83% (b) 56% (c) 67% (d) 71%

k e
138. What is ratio of the candidates who have a prior experience of working in Public Sector Banks in rural areas
only to the candidates who have a prior experience of working in Private Sector Banks in rural areas only?

n
(a) 4 : 3 (b) 3 : 2 (c) 2 : 3 (d) 3 : 4

(a) 28.53% (b) 28.73%

p r a
139. Candidates who have worked in Private Sector Banks in urban areas is what percent less than the number of
candidates who have worked in public sector banks in urban areas?
(c) 24.43% (d 26.53%

(a) 27:100
To
140. If 37% of inexperienced candidates are post graduate then find the ratio between total number of experienced
candidates to the number of inexperienced candidates who are not postgraduate?
(b) 100:27 (c) 100:63 (d) None of these

Directions (Q.141-Q.145): Study the following information carefully to answer the questions given below it.
In a school of 2500 students, all the students have enrolled for different games viz. hockey, table-tennis,
badminton, football, cricket, chess and carom. The respective ratio of girls to boys in the school is 3 : 2. 20% of
the boys play only cricket. 25% of the girls play table-tennis, badminton and carom only. 26.8% of the boys play
only football. The number of girls playing only cricket is 175%tr-5H7Gof the
4D3H boys
1K6I 4A playing the same. The respective
ratio of girls and boys playing only chess is 12 : 11. 25.7% of the boys play hockey, table-tennis and carom only.
One-fourth of the girls play only badminton. The remaining girls play football and hockey only. The remaining
boys play only chess.

141. How many students play more than one game? 6N4O
tr-5L7S4T3M1M
(a) 850 (b) 862 (c) 732 (d) 670

142. The total number of students playing hockey is what per cent of the total number of students in the school?
(a) 25.7% (b) 10.8% (c) 14.28% (d) 21.14%

143. What is the respective ratio of total number of boys playing chess to the total number of girls playing badminton?
(a) 11 : 30 b) 13 : 32 (c) 9 : 29 (d) 13 : 29

Head Office: 127, Zone II, MP Nagar, Bhopal |+91-7676564400| https://www.toprankers.com Page 40 of 44
144. What is the total number of students playing any 1 or more than 1 of the games out of football, cricket and table-
tennis?
(a) 1300 (b) 1550 (c) 1450 (d) 1650

145. How many students play carrom?


(a) 475 (b) 600 (c) 538 (d) None of these

Directions (Q.146-Q.150): There are 1000 students in a college. Out of 1000 students some appeared in exams
‘X’, ‘Y’ and ‘Z’ while some not. Number of student not appeared in any exam is equal to number of students
appeared in exam ‘Z’ only. Number of students appeared in exam ‘Y’ is 360. Ratio of number of students
appeared in exam ‘X’ and ‘Y’ only to number of students appeared in exam ‘Y’ and ‘Z’ only is 2 : 3. Number

m
of student appeared in exam ‘X’ and ‘Z’ both is half of number of students appeared in only exam ‘Z’.
Number of students appeared in exam ‘X’ only is 50% more than number of students appeared in ‘Y’ only.

c
of students appeared in ‘Y’ exam only is same as number of students appeared in ‘Y’ and ‘Z’ only.

. o
Number of students appeared in all the three exam is 4% of the total number of students in the college. Number

rs
146. How many students appeared in at least two exams?
(a) 240 (b) 260 (c) 300 (d) 360

147. How many students appeared in two exams only?

k e
n
(a) 280 (b) 220 (c) 340 (d) 260

148. How many students appeared in at most two exams?


(a) 240 (b) 260 (c) 300

p r a (d) 960

(b) 360

(b) 360 To
149. How many students not appeared in exam Y?
(a) 440 (c) 540

150. How many students appeared in exam X or in exam Z?


(a) 240 (c) 500
(d) 640

(d) 680

1K6I4A
tr-5H7G4D3H

6N4O
tr-5L7S4T3M1M

Head Office: 127, Zone II, MP Nagar, Bhopal |+91-7676564400| https://www.toprankers.com Page 41 of 44
Notes:-

c o m
rs .
k e
r an
o p
T
1K6I4A
tr-5H7G4D3H

6N4O
tr-5L7S4T3M1M

Head Office: 127, Zone II, MP Nagar, Bhopal |+91-7676564400| https://www.toprankers.com Page 42 of 44
Notes:-

c o m
rs .
k e
r an
o p
T
1K6I4A
tr-5H7G4D3H

6N4O
tr-5L7S4T3M1M

Head Office: 127, Zone II, MP Nagar, Bhopal |+91-7676564400| https://www.toprankers.com Page 43 of 44
Notes:-

c o m
rs .
k e
r an
o p
T
1K6I4A
tr-5H7G4D3H

6N4O
tr-5L7S4T3M1M

Head Office: 127, Zone II, MP Nagar, Bhopal |+91-7676564400| https://www.toprankers.com Page 44 of 44

You might also like